Clinical Rotation & Didactic Year Exams

Ace your homework & exams now with Quizwiz!

what is the most common pathogen of neonatal meningitis.

Streptococcus agalactiae

A 15-year-old male presents to the office with his mother with a complaint of constant itching and burning on his arms and thighs for 2 weeks. His mother admits to giving him acetaminophen without relief. The patient is taking no other medications. On clinical exam, there are multiple 2-cm wheals with a few small papules on his thighs and forearms. They are red and slightly raised. What is the diagnosis? A. Urticaria B. Erythema multiforme C. Erythema Ab Igne D. Erythema Nodosa E. Nummular eczema

*A. Urticaria*

tx for dawn phenomenon

- adding insulin to the evening or nighttime regimen.

what is the somogyi effect

- begins with an episode of nocturnal hypoglycemia - this triggers the body's couterregulatory response which causes elevated glucose levels in the morning

tx for pt experiencing the somogyi effect

- correcting the nocturnal hyperinsulinemia with a bedtime snack - limiting evening exercise, decreasing the dinner dose of aspart - decreasing the daily dose of glargine if his basal blood sugars, fasting, and pre-meal are low.

what is the dawn phenomenon?

- morning hyperglycemia due to the physiologic secretion of growth hormone in the early morning hours causing insulin resistance

the preferred marker(s) for detection of Acute MI at 3 - 6 hours after onset are: A. CK-MB2/CK-MB1 B. homocysteine C. myoglobin D. C-reactive protein E. troponin T and I

E. troponin T and I

Your patient is a 68 y/o male with a holosystolic, high-pitched, blowing murmur best heard at the cardiac apex with radiation to the axilla. What type of murmur does this patient have? A mitral valve regurgitation B pulmonic stenosis C tricuspid regurgitation D aortic regurgitation E mitral stenosis

A mitral valve regurgitation

which is correlated with a reduced risk for cardiovascular disease? A. BMI < 32 B. elevated serum HDL level C. elevated serum LDL level D. decreased serum torponin I level E. low hip to waist ratio

B. elevated serum HDL level

A puerpera is which of the following? A A woman who has just given birth B A parturient C A nullipara D Not a gravida E May be a primipara but not a multipara

A A woman who has just given birth

A 25-year-old male medical resident presents with 'acute red eyes' with copious watery discharge. He also complains of some aversion to bright light. He describes a sensation of a 'gritty' foreign body in his left eye, which he rubs continually. He denies any visual disturbance or pain. On physical exam the conjunctiva of both eyes are injected and mildly swollen. The eyelids are not swollen. Both eyes demonstrate normal and pain free motion. The tympanic membranes and pharynx are normal. There is no lymphadenopathy. At this time, the presence of bilateral conjunctival redness with watery discharge in this patient would have alerted you to A Allergic conjunctivitis B Bacterial conjunctivitis C Chlamydial conjunctivitis D Contact lens abuse E Hyperacute conjunctivitis

A Allergic conjunctivitis

A 52-year-old, female, is brought in by family members due to a change in her mental status. They state that she is a renal dialysis patient, and that her last dialysis was three days ago. She is due to go in today. She also has hypertension and CHF. When they saw her the previous night, she was in a normal state, but she was complaining that her hiatal hernia was bothering her. When they went to visit her today, she was very difficult to arouse. Two bottles of Mylanta (Antacid - Antiflatulent) were empty next to her. Paramedics brought her in, having had no response to Narcan (Naloxone hydrochloride injection), and a normal range Accucheck. On physical exam, there presents an obese female who is difficult to arouse, and speaks only garbled, unintelligible words. She is actively vomiting. Vital signs are as follows: 90 systolic; pulse 90; respirations 12; Temp 37.5° C (99.5 ° F). HEENT is essentially normal. Neck is supple. Lungs show basilar crackles. Heart shows a positive S4. Abdomen is obese with decreased bowel sounds, no localized tenderness or rebound. Extremities show 1+ pitting edema. Her A-V fistula is in her right forearm and has a positive thrill. Neurologic exam is non-focal, she moves all extremities in a non-purposeful fashion, however, deep tendon reflexes cannot be elicited. Initial laboratory studies show the following: Chem 7: Na 135; K 5.5; Cl 105; HCO3 27; Glucose 125; CBC, WBC:6.5 without a shift; H/H are 9/27. CT of the head is normal. CXR shows cardiomegaly with interstitial fluffiness. The best treatment for this patient would be A Arrangement for immediate dialysis B 2 grams MgSO4 IV over 1/2 hour C 500cc bolus .9 NaCl and 60 mg Lasix IV D 10% Calcium Chloride; 5 cc over 30 seconds E 200 cc of 10% glucose with 15 units of regular insulin over 30 minutes

A Arrangement for immediate dialysis

A 70-year-old male comes in with complaints of cough with productive sputum. A chest X-ray reveals several 1-4 cm mass lesions with air fluid levels on the right lung but not on the left. On the basis of the radiological findings the probable diagnosis would be A Aspiration B Septicemia C Bronchiectasis D Bronchopulmonary sequestration E Squamous cell carcinoma

A Aspiration

A 38-year-old patient of Mediterranean descent presents with mild, isolated microcytic anemia. His RBC count is normal, but peripheral blood smear shows target cells. He is otherwise healthy and takes no medications. Which of the following is the most likely diagnosis? A Beta-thalassemia minor B Beta-thalassemia major C Alpha-thalassemia D Anemia of chronic disease E Sideroblastic Anemia

A Beta-thalassemia minor

A 73-year old man presents after a 15-minute episode of right eye vision loss, which he described as being "like a shade being pulled down." Which of the following diagnostic tests is most likely to be abnormal? A Carotid ultrasound B Computed tomography C Electrocardiogram D Erythrocyte sedimentation rate E Electroencephalogram

A Carotid ultrasound

A 35-year-old male presents with history of headache. On examination the BP in the upper limbs is 180/100 mmHg and that of the lower limbs is 120/70 mmHg. On auscultation a continuous murmur is heard over the back. From the history and examination findings what is the most probable diagnosis in this patient? A Coarctation of the aorta B Takayasu's arteritis C Giant cell arteritis D Aortic stenosis E Dissection of the aorta

A Coarctation of the aorta

A 36-year-old African-American woman presents for an evaluation of a nonproductive cough, malaise, mild fever, and mild dyspnea. A chest X-ray demonstrates a right hilar mass. A pulmonologist is consulted and performs a biopsy during bronchoscopy. The report reveals that the mass is a noncaseating granuloma. Which of the following therapeutics do you prescribe? A Corticosteroids B Inhaled Beta agonists C Anti-tumor necrosis factor (Anti-TNF) D Amphotericin B E Cyclosporine

A Corticosteroids

A 78-year-old Caucasian female presents with severe stiffness, pain, and fatigue involving the shoulders and thighs. She states her symptoms are worse in the mornings. She has a history of osteoarthritis, but states this has felt very different. She also complains of severe headaches for the last several days. Exam reveals an appropriate gait, though it is noted that the patient has difficulty rising from the exam chair and table without considerable effort and use of her upper extremities. Full musculoskeletal exam is significant for 4/5 strength of all motions of the shoulders and hips bilaterally. Strength for all other motions is 5/5. Full neurological testing including cranial nerves is within normal limits otherwise and intact, respectively. An ESR and temporal biopsy is ordered. Given the most likely diagnosis, the first-line treatment indicated is: A Corticosteroids B IV antibiotics C Thrombolytics D NSAIDs E Anticonvulsants

A Corticosteroids

You have a 70 y/o female interested in getting vaccinated for herpes zoster. Which of the following would be a contraindication for receiving herpes zoster vaccination? A Corticosteroids prescribed currently B Chemotherapy received in the past C Never received varicella vaccination D History of surgical or functional asplenia E Previous herpes zoster infection

A Corticosteroids prescribed currently

A 30-year-old female presents to the emergency room complaining of tingling sensations around her mouth. On examination, her temperature is 37C, BP 120/60. In addition, you notice flexing of the metacarpal joints and adduction of the thumb when the BP cuff is left on for too long. Her mental status examination is normal. Given the most likely diagnosis, what testing is indicated next in her management? A Electrocardiogram B MRI without contrast of the brain C Whole-body PET scan D DEXA scan E Thyroid function testing

A Electrocardiogram

Mr. Jones, a 56-year-old male, presents with a chief complaint of abdominal pain, nausea and vomiting, fever, and clammy skin for the past 24 hours. Home treatment included acetaminophen and fluids with little relief. His past medical history includes gallstones and congestive heart disease. His medications include thiazide diuretics. On clinical exam, his skin is cool and clammy, abdominal tenderness in the upper quadrants, heart rate 100, and blood pressure is 110/70. What is your treatment? A Fluid replacement B Reduced-fat diet C Laparotomy D Chemotherapy E Surgery

A Fluid replacement

A 40-year-old male presents with complaints of pain while swallowing for the past week. Sharply circumscribed ulcers are seen on upper endoscopy in the lower esophagus of this patient known to be HIV positive with a CD4 lymphocyte count of 150/microliter. These ulcers are probably due to: A Herpes simplex B Candida albicans C Helicobacter pylori D Cytomegalovirus E Mycobacterium-avium complex

A Herpes simplex

A 21-year-old woman presents with recurrent painful nodules that form in her armpits. On physical examination, you note red inflammatory nodules that are very tender to palpation. Also noted are open comedomes that seem to be paired. The patient indicates that these areas ultimately break down and drain a foul-smelling, purulent material. What is the most likely diagnosis? A Hidradenitis suppurativa B Roth spots C Sebaceous gland hyperplasia D Cellulitis E Acanthosis nigricans

A Hidradenitis suppurativa

A 25-year-old bartender at a country club challenges his buddies to a breath-holding contest in the pool. When the others come up for air they note that he is still floating face down. He cannot be resuscitated. His autopsy (post-mortem examination) is most likely to reveal which of the following? A Hypertrophic cardiomyopathy B Narrowing of his left main coronary artery C Narrowing of his left anterior descending coronary artery D Narrowing of his right coronary artery E Normal cardiovascular anatomy

A Hypertrophic cardiomyopathy

A 2-year-old boy with a history of sickle cell disease who is on penicillin prophylaxis presents for routine check up. The child is asymptomatic and appears healthy. He has received a regular schedule of immunizations to date. What additional vaccination(s) needs to be administered in his case? A Influenza, pneumococcal, and meningococcal vaccine B DTaP and Hepatitis B vaccine C IPV and varicella vaccine D Tdap, MMR, and Hepatitis B vaccine E Only influenza vaccine

A Influenza, pneumococcal, and meningococcal vaccine

A 56-year-old obese male is presents to the office with complaints of foamy, sweet-smelling urine. He also admits polyphagia and polydipsia. Work-up ultimately reveals diabetes mellitus with a random serum glucose of 320 mg/dL and an A1C of 9.0%. According to current guidelines, what pharmacological therapy is recommended for this patient? A Insulin B Metformin C Glyburide and metformin D Liraglutide (Victoza) and glyburide E Sitagliptin (Januvia)

A Insulin

Which lab results would indicate secondary hypothyroidism? A Low TSH and low free T4 B Low TSH and high free T4 C High TSH and low free T4 D High TSH and high free T4 E Normal TSH and low free T4

A Low TSH and low free T4

Mrs. Jones is a 43-year-old female with a chief complaint of pain and stiffness in both wrists and knees. The pain developed slowly and started in the groin and on the outside of the hips and radiates sometimes to the buttocks and knee. The symptoms in the lower limb have been present for 6 months resulting in the patient walking with a limp. The wrist pain is a new presentation. Upon physical exam, there is no joint swelling or erythema of the wrists or knees. Some crepitus is noted with range of motion of both knees (right more than left). What is the most likely diagnosis? A Osteoarthritis B Rheumatoid arthritis C Gouty arthritis D Calcium pyrophosphate arthropathy E Reactive arthritis

A Osteoarthritis

A 48-year-old woman was admitted to the hospital for hysterectomy for multiple uterine fibroids. In 1980 she had undergone a left nephrectomy for pyelolithiasis and three years later a parathyroid adenoma was discovered during investigation for hypercalcemia. During her admission for subtotal parathyroidectomy, the patient complained of recurrent epigastric pain, and investigation of this revealed severe peptic ulceration associated with a raised gastrin level of 810 pg/ml (normal less than 95 pg/ml). The patient was subsequently asymptomatic but continued with regular endocrinological outpatient review. The diagnosis of Zollinger-Ellison syndrome was made and a total gastrectomy and distal pancreatectomy were performed. During the operation the pancreatic adenoma was removed, substantiating the diagnosis of Zollinger-Ellison syndrome. The final diagnosis was a multiple endocrine neoplasia 1 (MEN-1), because gastrinomas are the most common pancreatic tumors associated with MEN-1. According to this diagnosis, which organ should be particularly controlled? A Pituitary B Liver C Lung D Muscles E Heart

A Pituitary

A 70-year-old black male was first seen 15 years ago. At that time he presented because of a history of an elevated blood count and splenomegaly. Both of these findings had been discovered at the time of a routine physical examination. He recalled that he had had mild right frontal headaches for many years. His physician had requested the phlebotomy, and he had five units of blood removed over nine months. With that treatment he felt that his problem had lessened. Past history had revealed no significant illness or operations. Social history showed that he had never been a smoker. He worked as bank clerk, and he has never resided at high altitude. Family history revealed no individuals who had elevated blood counts. The most likely diagnosis is: A Polycythemia vera B Relative polycythemia C Secondary polycythemia D Autonomous erythropoietin production (tumor) E Systemic hypoxia - decreased arterial oxygen tension caused by pulmonary disease

A Polycythemia vera

A 33-year-old woman presents to your office with a 1 year history of a combination of major depression and manic episodes. She also experiences social withdrawal, delusions and auditory hallucinations. Which of the following is the most likely diagnosis? A Schizoaffective disorder B Schizophreniform disorder C Delusional disorder D Brief Psychotic Disorder E Psychotic Disorder NOS

A Schizoaffective disorder

A 20-year-old woman is well-known to your practice. On this visit she complains of abdominal pain. In the past she has sought treatment for head, back, joint, and chest pain, she has complained of nausea and vomiting, she has a history of irregular menses, and she has complained of localized weakness to her right leg. Her symptoms have never been fully explained and you rule out such diagnoses such a Factitious disorder and Malingering. This woman most likely suffers from (a): A Somatization disorder B Body dysmorphic disorder C Conversion disorder D Hypochondriasis E Somatoform pain disorder

A Somatization disorder

A 72 year old female presents with 6 months of gradually worsening right shoulder pain. She has noticed that she is unable to perform overhead movements with the right arm over the last few weeks, and she feels like her arm is weak when she tries to lift it to brush her hair. On exam, the shoulders bilaterally are without redness, swelling, or obvious deformity, however, she is unable to move the right arm beyond 60 degrees of abduction. She is unable to complete the empty-can test. A right rotator cuff tear is diagnosed. Which rotator cuff muscle is most likely affected based on her history and physical exam? A Supraspinatus B Infraspinatus C Teres minor D Subscapularis E Trapezius

A Supraspinatus

A 40-year-old female diagnosed with acute pericarditis comes to your office complaining of chest pain that is retrosternal with no radiation, stitching in character, increases with inspiration and relieved by leaning forward. She had a common cold two weeks ago. On examination, she looks healthy, with temperature =38.3 c, blood pressure =110/70, HR=96 B/M and normal respiratory rate. A scratchy triphasic sound is heard inside the apex and normal chest auscultation. Normal chest x-ray and ECG shows generalized ST segment elevation. Echocardiography shows normal LV function with normal contraction and mild pericardial effusion. What is the most common cause of pericarditis in this patient? A Viral infection B Tuberculosis infection C Bacterial infection D Fungal infection E Neoplasm

A Viral infection

Dysphagia, dysarthria, headache, and psychiatric symptoms of bipolar disorder may be diagnosed as a true psychiatric disorder, but the diagnosis may also require further investigation. A 27-year-old woman presents with these symptoms and repeated spontaneous abortions and periods of amenorrhea. During her exam, the physician notes prominent brown rings at the outer edge of the irides bilaterally. The physician proceeds with workup to determine if there is a diagnosis of: A Wilson's Disease B Hemachromatosis C Kayser-Fleischer D Pickwickian syndrome E Pulmonary hypertension

A Wilson's Disease

A 48-year-old alcoholic patient with ascites has been treated on a long-term basis with spironolactone. The ECG shows peaked T-waves, flattening of the P-wave, and prolongation of the PR interval. You expect to see the following laboratory findings: A hyponatremia and hyperkalemia B eunatremia and eukalemia C hypernatremia and hypokalemia D hyponatremia and hypokalemia E eunatremia and hyperkalemia

A hyponatremia and hyperkalemia

A 24-year-old woman with acute uveitis and erythema nodosum is found to have bilateral symmetric hilar adenopathy and paratracheal adenopathy without chest symptoms. The most likely diagnosis is A. Tuberculosis B. AIDS C. Lyme disease D. Sarcoidosis E. Rheumatoid arthritis

D. Sarcoidosis

Your patient is a 16-year-old female who is brought to your office by her concerned mother who states that her daughter has not "started her periods." History includes normal childhood development with no major illnesses. There is an absence of facial hair. Pelvic examination reveals a short, blind vagina. You suspect testicular feminization syndrome. Which of the following additionally characterize testicular feminization syndrome? A normal male karyotype B normal androgen receptor protein C absence of wolffian ducts D excess pubic hair E mammary aplasia

A normal male karyotype

A 30-year-old woman presents to the hospital with a 2-week history of difficulty with swallowing food. She is not on any medications. She is a single secretary who drinks alcohol over the weekends. The systemic examination is essentially normal. A barium swallow reveals bird beaking. Manometry shows increased lower esophageal sphincter pressures. A. Achalasia B. Diffuse esophageal spasm C. Globus hystericus D. Zenkers diverticulum E. Scleroderma

A. Achalasia Achalasia is a motility disorder of the esophagus characterized by impaired esophageal body peristalsis and failure of the lower esophageal sphincter (LES) to relax during deglutition. Patients usually present with dysphagia for both solids and liquids. They may also complain of a nocturnal cough and regurgitation of foodstuffs. Barium swallow reveals dilatation of the proximal esophagus with narrowing of the distal esophageal area and the bird beaking appearance. Patients with scleroderma who develop dysphagia as a result of impaired esophageal motility complain of dysphagia for both solids and liquids. Barium swallow shows dilatation of the lower esophagus with poor sphincter tone. Zenker's diverticulum, also known as pharyngoesophageal diverticulum, is a herniation of the esophageal mucosa through the cricopharyngeal muscle.

A 56-year-old man presents with a 24-hour history of abdominal pain, nausea and vomiting, fever, and clammy skin. Home treatment included acetaminophen and fluids; treatment offered little relief. His past medical history includes gallstones and congestive heart failure. On clinical exam, his skin is cool and clammy, he has abdominal tenderness in the upper quadrants, his heart rate is 100, and his blood pressure is 110/70. What is his most likely diagnosis? A Acute pancreatitis B Chronic pancreatitis C Pancreatic carcinoma D Diverticulitis E Gastric carcinoma

A. Acute pancreatitis

A 25-year-old male is brought to the hospital after a vehicular accident. He is intubated and placed on a ventilator. He becomes progressively difficult to oxygenate despite increasing the PEEP and the oxygen supply to 100%. Patient remains afebrile. He dies several days later. At autopsy, the lung shows diffuse hyaline membranes in the alveoli, thickened alveolar walls and many alveolar macrophages but few neutrophils. These findings suggest that the patient suffered from A. Adult respiratory distress syndrome B. Bronchopneumonia C. Chronic bronchitis D. Bronchiectasis E. Viral pneumonia

A. Adult respiratory distress syndrome - It characteristically develops following lung injury from a variety of serious illnesses or accidents. Some common conditions predisposing to ARDS are: a) Gram-negative septicemia b) multiple trauma c) pneumonia (bacterial, viral, Pneumocystis, mycoplasma) d) severe burns e) gastric aspirations f) perforated viscus - Bronchopneumonia, if develops as a nosocomial infection, is likely to be accompanied by signs of sepsis like fever and neutrophilic infiltration in the alveoli. - Chronic bronchitis develops and continues over months to years and probably is due to environmental ages. Multiple trauma due to an accident is an unlikely cause for chronic bronchitis. - Bronchiectasis is a process that takes at least weeks to months to develop and the cause is obstruction of airways in one or more part of the lung.

A 48-year-old African American female presents to your office for follow up of her diarrhea-variety irritable bowel syndrome. She has been having worsening of her symptoms over the past few week, and admits to increased stress levels and difficulty sleeping at night due to anxiety. Past medical history positive for mild premenstrual dysphoric disorder for which she has tried sertraline (Zoloft) and fluoxetine (Prozac); both were discontinued due to headaches and worsening diarrhea. Abdominal exam is benign. She is requesting that you give her something to help her symptoms. Which of the following medications is the best choice for her to try? A Amitriptyline at bedtime B Acetaminophen with codeine as needed C Amitriptyline as needed D Paroxetine at bedtime E Lorazepam as needed

A. Amitriptyline at bedtime

An 33-year-old male with ankylosing spondylitis presents to your office with weakness and pallor. His vital signs are pulse 85/min, BP 120/80, temperature 98.9. His physical examination shows pallor in the mucous membranes. His CBC and red cell indices are as follows: RBC 4.0X106/ml Hemoglobin 10.5gm/dl HCT 32% MCV 80 fl MCH 26 pg MCHC 33 gm/dl WBC 10.0x 103/ml Platelets 200x103/ml Total iron binding capacity 300 ug/dl Serum iron 30 ug/dl Transferrin % saturation 10% The morphologic review of the blood smear shows essentially normal morphology. Refer to the case. The cause of the patient's anemia is: A. Anemia of chronic disease B. Hemoglobinopathy C. Hemolytic anemia D. Iron deficiency anemia E. Leukemia

A. Anemia of chronic disease

In inversion injury ankle sprains, which of the following ligaments is most likely to be damaged? A Anterior talofibular ligament B Posterior talofibular ligament C Calcaneofibular ligament D Anterior tibiofibular ligament E Deltoid ligament

A. Anterior talofibular ligament

A 5-year-old male is involved in a high speed MVA and sustains a blunt trauma to his head and right leg. On arrival to the emergency department, the patient is boarded with a c-collar in place. He is moaning incomprehensible words, but not responding to verbal stimuli. He does not open his eyes at all. He is exhibiting flexion of his upper extremities. He has an obvious deformity of his right lower extremity. His vital signs are as follows: Temp 37.8, RR 10 (periodic breathing), Pulse120, BP 100/80. What is your initial management of the above patient? A. Assess the airway and make sure that it is intact B. Insert 2 large-bore IV's and give large volumes of fluids C. Splint the right leg D. Obtain an emergent CT of the head E. Immediately obtain cervical films to assess for any neck injuries

A. Assess the airway and make sure that it is intact

A 55-year-old African American male is brought to the emergency room, for shortness of breath and palpitations. He has a history of myocardial infarction, 10 months ago, and several years of hypertension. He has been a heavy smoker (more then 30 cigarettes a day) for more than 30 years. His pulse is irregularly irregular, 150/min. The most likely ECG diagnosis will be: A. Atrial flutter B. First degree AV block C. Complete AV block D. Atrial fibrillation E. Supraventricular tachycardia

A. Atrial flutter

55 yo male comes to ED with severe SOB. CXR shows pleural effusion. Thoracentesis reveals transudative fluid. What is the most likely dx? A. CHF B. constrictive pericarditis C. sarcoidosis D. adenocarcinoma of lung E. empyema

A. CHF

A 28-year-old male comes to your office seeking advice. He suffers from diarrhea in the form of watery unformed stools, which appeared two months ago. In addition, he has an occasional appearance of mild pains in left periumbilical and left iliac regions, which are not meal-related. He gives personal history of an established diagnosis of Berger's disease (IgA nephropathy) at the age of 26; at the same time, he was treated for iritis by the ophthalmologist. He also states that he was treated for perianal fistula three months ago. The remainder of his personal history is unremarkable. Physical examination reveals a moderately nourished patient, as well as minimal tenderness that is present left, and below the umbilicus. The remainder of his general physical finding is unremarkable. Laboratory analyses reveal an elevated erythrocyte sedimentation rate (53mm/hour), slight hypoproteinemia (serum protein level 60g/l, serum albumin level 33g/l) and proteinuria (24-hour urinary protein loss is 0.7 grams, in accordance with the diagnosis of Berger's disease). There is also a mild anemia (total erythrocyte count 3.9x10/6 erythrocytes/ml, hemoglobin level l2.1 g/dL). Other routine laboratory analyses reveal no abnormalities. Stool culture does not reveal the presence of pathogenic bacteria, and the results of the search for parasites and their ova in the stool are negative. Refer to the case. Which of the following diagnostic methods is most likely to reveal the cause of diarrhea? A. Colonoscopy with biopsies B. Gastroscopy and biopsy C. Performance of jejunal biopsy D. Determination of fecal fat excretion E. Barium small bowel examination

A. Colonoscopy with biopsies

A 12 year old boy presents to the emergency department with a six hour history of increasing abdominal pain, fever and anorexia. The patient admits nausea, but denies vomiting or diarrhea. He reports the pain initially began in the periumbilical region, but on exam he is most tender in the right lower quadrant. CBC with differential is remarkable for a WBC of 14,000. The best course of action now is to: A. Contact the surgeon for evaluation of appendicitis B. Start him on oral antibiotics and advise him to return should his symptoms fail to improve C. Admit him to the hospital and start him on intravenous antibiotics D. Obtain a CT scan of his abdomen and pelvis E. Obtain X-rays of his abdomen

A. Contact the surgeon for evaluation of appendicitis

A 32-year-old massage therapist is diagnosed with breast cancer. Her mother and maternal grandmother both died of metastatic breast cancer, initially diagnosed in their early 30's. She undergoes a mastectomy with axillary node dissection and neoadjuvant chemotherapy. She then agrees to complete post-mastectomy x-irradiation because 2 of her 25 axillary lymph nodes were positive for metastatic breast cancer. Afterward, she is cleared to return to work and her physical examination reveals no pathologic findings. Several months later, she presents to your care. She describes waking with sudden inability to walk and loss of bowel and bladder control. Physical exam reveals a spastic paraparesis of both of her lower extremities with hyperreflexia. She has a present Babinski reflex bilaterally. She has no sensation below the waist. She is alert with clear and appropriate speech. The motor and sensory testing of her upper extremities is intact. She specifically denies any recent or remote trauma. What is the most likely cause of her acute spastic paraparesis? A Cord compression B Amyotrophic lateral sclerosis C Malingering D Syringomyelia E Catatonic schizophrenia

A. Cord compression

Foot drop and a high-stepping gate are the result of damage to what nerve? A. Deep peroneal nerve B. Superficial peroneal nerve C. Tibial nerve D. Sciatic nerve E. Obturator nerve

A. Deep peroneal nerve - superficial peroneal nerve innervates the lateral peroneals (evert the foot) - The tibial nerve innervates posterior muscles of the leg - The sciatic nerve, while giving rise to the superficial and deep peroneal nerves, also innervates the hip (extension), resulting in excessive hip flexion for the foot to clear the ground - The obturator nerve innervates the adductor muscles.

A 50-year-old woman has trouble making any decisions in her life without asking others what she should do. She cannot assume responsibility for anything. She fears that disagreeing with others will cause her lose their approval. She has trouble doing things on her own because she feels she is incompetent and stupid. She will volunteer to do things that hurt her in order to receive nurturing and support. She cannot be alone because she feels she needs someone to take care of her. When she got sick and was in the hospital, the nurses objected to the fact that she was very demanding and complaining all the time. What type of personality disorder does this female have? A. Dependent personality disorder B. Avoidant personality disorder C. Narcissistic personality disorder D. Borderline personality disorder E. Antisocial personality disorder

A. Dependent personality disorder

A previously healthy 26-year-old female presents to the ER with a swollen cyanotic left calf. Her vital signs are as follows: pulse 80/min, BP 120/80, temperature 98.1F. Doppler ultrasound shows reduced flow in the left deep Saphenous vein. Her CBC and Coagulation results are as follows: vein. RBC 4.9X106/ml Hemoglobin 12.9mg/dl WBC 9.8x 103/ml Platelets 180x103/ml Bleeding time 7 minutes PT 12 seconds PTT 32 seconds TT 2 seconds longer than control Fibrinogen 290 mg/dl Fibrin degradation products 20 ug/ml D-dimer 110 ng/ml PTT without APC 38 seconds PTT with APC 67 seconds Protein C activity 20% Protein S activity 80% Antithrombin III activity 90% Refer to the case. The patient started complaining about red painful multiple skin lesions 4 days after the initiation of Warfarin therapy. The best therapeutic approach at this point would comprise: A Discontinuation of Warfarin and administration of Vitamin-K injections, heparin, and Protein C concentrate B Rechecking coagulation labs in 24 hours C Fresh frozen plasma infusion with discontinuation of Warfarin D Maintaining the patient on Warfarin and initiating protein C concentrate infusion E Protein C concentrate infusion with discontinuation of Warfarin

A. Discontinuation of Warfarin and administration of Vitamin-K injections, heparin, and Protein C concentrate

A 25-year-old white female comes into your office. She states that she and her husband have been trying to have a baby for the last two years. The patients' medical history included cyclical pelvic pain, dysmenorrhea and dyspareunia. The physical exam reveals the following: diffuse abdominal or pelvic pain of variable location, nodular thickening and tenderness along the uterosacral ligaments, on the posterior surface of the uterus, and in the posterior cul-de-sac, scarring and narrowing of the posterior vaginal fornix, and adnexal enlargement and tenderness. What is the most likely diagnosis? A. Endometriosis B. Ectopic pregnancy C. Adnexal mass D. Pelvic relaxation E. Leiomyomatous uterus

A. Endometriosis

A 43-year-old female complains of food sticking in her throat. She sees you with her complaints. Her history is unremarkable. Further tests are done, and she is found to have small pockets in her esophagus. What is the correct terminology for this condition? A. Esophageal diverticula B Esophageal web C Esophageal laceration D Esophagitis E Mallory-Weiss tear

A. Esophageal diverticua

A 26-year-old female is found positive for occult blood in her stool sample. A colonoscopy is performed. It shows numerous polyps, numbering more than 200, ranging in size from 1 cm - 5 cms. One polyp is resected. Histologically, the polyp shows the features of tubular adenoma. There is no invasion. Molecular analysis of patient's fibroblasts shows mutation in APC gene. The most likely diagnosis in this case is A Familial adenomatous Polyp B Juvenile Polyp C Inflammatory Polyp D Peutz-Jeghers Polyp E Lymphoid Polyp

A. Familial adenomatous Polyp

Which of the following is the most common and most aggressive type of primary brain tumor in adults? A. Glioblastoma multiforme B. Ependymoma C. Medulloblastoma D. Oligodendroglioma E. Schwannoma

A. Glioblastoma multiforme

A 5-year-old boy is brought to the pediatric outpatient department for complaints of recurrent episodes of high fever, headache, and vomiting. The boy is playful and active when free of fever. He had been suffering from cough and cold for the past few days. On examination, the physician notices inflammation and enlargement of palatine tonsils bilaterally, with congestion of the posterior pharyngeal wall (refer to the image). The child gags as the tonsil is touched with the instrument. The presence of this reflex proves the integrity of which nerve? A. Glossopharyngeal nerve B. Facial nerve C. Oculomotor nerve D. Accessory nerve E. Trigeminal nerve

A. Glossopharyngeal nerve

Mrs. Jones presents with painful distal phalangeal joints (DIPJ) in both hands. Past medical history is positive for a 10-year history of arthritis that is treated with NSAIDs. On clinical exam, both hands present with enlargements of the DIPJs that are not warm or red but are tender to palpation. This condition is called what? A. Heberden's node B. Bouchard's node C. Mucinous cyst D. Gout E. Inflamed bursa

A. Heberden's node

An 86-year-old woman presents to her PCP with recent onset of intractable headaches, jaw claudication, and visual field changes, including diplopia. Her past medical history is significant for Polymyalgia Rheumatica. Based on the most likely diagnosis, what is the primary reason for prompt diagnosis and treatment of this condition? A. Prevent blindness B. Prevent stroke C. Prevent facial palsy D. Prevent myocardial infarction E. Prevent hemorrhage

A. Prevent blindness Treatment consists of prednisone, 60mg given daily, immediately and a temporal artery biopsy performed after treatment initiated. Caused by occlusive arteritis of the ophthalmic artery

A 53-year-old female schoolteacher, had a subtotal thyroidectomy due to a thyroid adenoma, which was not responding to thyrostatic as well as radioactive iodine therapy. Surgery went well and the patient was discharged the day after the operation. She returns after two weeks complaining of hoarseness and a weak, faint voice since she had the surgery. Laryngoscopy shows reduce voice cord movement on the left, especially in abduction. What is the most likely cause? A. Iatrogenic nerve damage B. Scar tissue C. Hematoma D. Inflammation E. Luxation of arytenoid cartilages

A. Iatrogenic nerve damage

A 19-year-old woman presents with weakness and increased fatigue over the past several months. Laboratory studies reveal a hemoglobin of 9.7g/dL with a hematocrit of 32%. Which of the following results would indicate an iron deficiency anemia? A. Increased TIBC, decreased serum ferritin, decreased serum iron B. Decreased TIBC, decreased serum ferritin, decreased serum iron C. Increased TIBC, increased serum ferritin, increased serum iron D. Increased TIBC, increased serum ferritin, decreased serum iron E. Decreased TIBC, increased serum ferritin, decreased serum iron

A. Increased TIBC, decreased serum ferritin, decreased serum iron

Wound healing occurs in three phases: the inflammatory phase, the proliferative and repair phase, and the remodeling phase. What occurs in the proliferative and repair phase? A. Neovascularization B. Platelet degranulation C. Blood clotting D. Invasion by neutrophils E. Degradation of scar matrix by collagenase

A. Neovascularization Explanation The initial inflammatory phase of wound healing lasts for several days and is initiated by the processes of blood clotting and platelet degranulation. The alpha granules of the platelets release several growth factors which diffuse from the wound and permit the inflammatory cells to be chemotactically drawn into the injured area. Neutrophils and macrophages destroy bacteria and release proteases that degrade damaged extracellular matrix components while secreting additional growth factors. The inflammatory cells in the wound begin to decrease and the fibroblasts, endothelial cells, and the keratinocytes begin to increase and herald the entry of the proliferative and repair phase of wound healing. Neovascularization occurs in the proliferative and repair phase of wound healing. This is followed by the remodeling phase which can last for many months. During this last phase, a balance is reached between the synthesis of new components of the scar matrix and their degradation by metalloproteinases such as collagenase, gelatinase, and stromelysin.

A 45-year-old woman presents with yellow discoloration of the skin, clay colored stool, and dark urine. She is obese, but she is otherwise healthy. She also has long standing digestive troubles and an intolerance to fatty meals, for which she takes antispasmodic medications. On examination, there was mild tenderness in the upper right abdominal quadrant; serum bilirubin showed elevated total and conjugated types and normal unconjugated bilirubin. What is the most likely cause of her jaundice? A Obstructive reasons B Hemolytic anemia C Hepatic enzyme deficiency D Hepatitis E Drug-induced hemolysis

A. Obstructive reasons

A 28-year-old woman presents to the ER complaining that her periods have stopped for the last 2 months. She is worried that she may be pregnant but is concerned because she states that she and her boyfriend broke up about 2 months ago, and doesn't want to be a single mom. She also has noted a discharge from both breasts. You order a urinary HCG, which is negative. Which of the following diagnostic studies would you order next? A Prolactin level B Free T4 level C Pelvic ultrasound D Thyroid ultrasound E Serum calcium

A. Prolactin level

A newborn infant begins vomiting 4 weeks after birth. The mother states that her infant nurses well, but vomits forcefully after feeding. He has begun to lose weight. This is her first child and she is concerned. On examination, you note a palpable 2 cm firm mass in the abdomen. What is the most likely diagnosis? A. Pyloric stenosis B. Congenital megacolon C. Esophageal atresia D. Duodenal atresia E. Tracheoesophageal fistula

A. Pyloric stenosis

A 37-year-old male health care worker is admitted to the hospital after being diagnosed with tuberculosis. Before treatment is started, you inform him about the most common ways of transmitting the tubercle bacillus, as well as the medication used: isoniazid, rifampin, and pyrazinamide. Which of the following is a common side effect of rifampin therapy? A. Reddish Bodily Fluids B. Hyperuricemia C. Polyneuropathy D. Jaundice E. Muscle pain

A. Reddish Bodily Fluids

A 41-year-old woman has a three-month continuous history of morning stiffness and pain and swelling of the second and third metacarpophalangeal joints of both hands. The most likely diagnosis is: A Rheumatoid arthritis B Possible scleroderma C Probable systematic lupus erythematosus D Osteoarthritis E Classic psoriatic arthritis

A. Rheumatoid arthritis

A 43-year-old patient presents with an acute onset of palpitations and dyspnea for the past 3 hours. There is no other significant medical history. Chest X-ray and echocardiography are normal. EKG performed in the ER reveals absence of P wave with variable R-R interval with atrial rate of 300/min and ventricular rate of 120/min. Vitals: patient is afebrile, radial pulse- 90/min, BP- 110/70 mm Hg and RR- 18/min. A diagnosis of newly detected atrial fibrillation is made. What is the next course of action? A. Start diltiazem therapy B. Start propafenone C. Observation only D. AV(atrioventricular) nodal ablation E. Start amiodarone therapy

A. Start diltiazem therapy - The initial management in a hemodynamically stable (no hypotension or angina ) patient with atrial fibrillation involves control of ventricular rate so administer diltiazem (a rate-controlling drug) - If there is spontaneous conversion to sinus rhythm then the patient can be discharged after assessment for the cause of AF. Otherwise, the patient requires anticoagulation and electrical/chemical cardioversion at a later date such as amiodarone and propafenone - AV nodal ablation and permanent pacemaker implantation are highly effective in improving symptoms in patients with AF experiencing symptoms related to a rapid ventricular rate during AF, which cannot be adequately controlled with anti-arrhythmic or negative chronotropic medications.

A 22-year-old nulligravida whose last menstrual period was 2 weeks ago presents to your office with a 3-day history of profuse malodorous gray vaginal discharge. She has also noted mild pelvic pain during the same period of time. She is sexually active with 2 male partners and uses condoms occasionally. On wet mount, motile protozoans and white blood cells are noted. Which of the following may be seen with this type of vaginitis? A. Strawberry cervix B. Pseudohyphae C. Ferning D. Positive whiff test E. Clue cells on wet mount

A. Strawberry cervix - Pt has trichomoniasis, caused by Trichomonas vaginalis; profuse discharge with mild pelvic pain and/or dyspareunia; strawberry cervix in 10% of patients - Pseudohyphae may be seen in wet mount examinations of vulvovaginal candidiasis. - Ferning may be seen in microscopical examination of vaginal secretions in a pregnant patient with ruptured membranes. - A positive whiff test is noted with bacterial vaginosis are mixed with potassium hydroxide (KOH), liberating amines, which have a characteristic fishy odor. The whiff test is one of the 4 diagnostic criteria of the diagnosis of bacterial vaginosis

Part of the scapulohumeral or "SITS" muscle group of the rotator cuff, this muscle inserts on the greater tubercle and runs above the glenohumeral joint. What is it called? A. Supraspinatus B. Infraspinatus C. Teres minor D. Teres major E. Subscapularis

A. Supraspinatus - The infraspinatus and teres minor are muscles that also insert in the greater tubercle but cross behind the glenohumeral joint - the subscapularis inserts on the lesser tubercle

A 30-year-old Japanese female is brought to the clinic complaining of dizziness and weakness. On examination, her blood pressure is 90/60 mm Hg. Her upper extremities are cold. Both her radial pulses are very weak. On further questioning her relative says that she has had such several attacks in the past. The angiography reveals narrowing of the brachiocephalic and subclavian arteries. The most likely diagnosis in this case is A Takayasu's arteritis B Giant cell arteritis C Kawasaki disease D Microscopic polyangiitis E Polyarteritis nodosa

A. Takayasu's arteritis

A 32 year old male presents to the emergency room with fever, dry skin, abdominal distention and abdominal pain. His medical history is significant for ulcerative colitis. Given his history and physical exam findings, what is the most likely diagnosis? A. Toxic megacolon B. Small bowel obstruction C. Mesenteric ischemia D. Acute appendicitis E. Ileitis

A. Toxic megacolon

Which of the following hypersensitivity reactions is classical for extrinsic asthma? A Type I hypersensitivity reaction B Type II hypersensitivity reaction C Type III hypersensitivity reaction D Type IV hypersensitivity reaction E Neutrophilic reaction

A. Type I hypersensitivity reaction

A 24-year-old man presents with a mass in his left testicle. He said he noticed it the other day when he was performing a self-exam on himself. You palpate his scrotum and note a "bag of worms" consistency in his left hemiscrotum. What condition is this is consistent with? A. Varicocele B. Hydrocele C. Paraphimosis D. Spermatocele E. Balantis

A. Varicocele - A varicocele is a collection of dilated and tortuous veins surrounding the spermatic cord in the scrotum. - A hydrocele is a fluid collection in the scrotal space. Clinical features are scrotal enlargement, scrotal heaviness, and back pain. - Paraphimosis is a condition where the foreskin gets trapped behind the glans of the penis. - Spermatocele is a cyst found on the rete testes or the head of the epididymis. - Balantis is an inflammation of the superficial tissues of the glans penis

A 25-year-old man came to the outpatient clinic complaining of redness, itching, and swelling in the right upper eyelid that started 1 day ago. The redness increased, and there has been no eye discharge or watering. On examination, a pustular swelling at the root of the lashes was found. It is surrounded by redness, and it is tender on palpation. The lesion is shown in the following picture. A. Warm compresses and local antibiotic cream B. Local antifungal cream C. Systemic antibiotics D. Antiviral cream E. Systemic antiviral drug

A. Warm compresses and local antibiotic cream - There are 2 types of hordeolum: external hordeolum or acutely inflamed stye is caused by infection of gland of Zeis (sebaceous glands near the eyelashes) or Moll (apocrine glands of the lid margin). *Tx with warm compress*

76 yo previously healthy woman comes to ED with complaint of sudden onset weakness in R arm in the absence of any precipitating event. PE of the R upper extremity reveals pallor, coolness or right arm compared to left, absent brachial and radial pulses, a positive allen test, and decreased sensation at the hand and fingertips. Which of the following is the most common underlying cause of the patient's arm condition? A. a fib B. bacterial endocarditis C. myocardial infarction D. compartment syndrome E. Rheumatic valvular heart disease

A. a fib

Dx of staphylococus aureus pneumonia is made in a 90 yo woman. She receives a course of antibiotic therapy. 2 weeks later she no loner has a productive cough, but she still has a fever. A CXR reveals a 3 cm rounded density in the Right lower lobe whose contents form a central air-fluid level. There are no surrounding infiltrates. Which of the following it the best description for this outcome of her pneumonia? A. abscess formation B. caseating necrosis C bulla formation D. lung infarction E. bonchiectasis

A. abscess formation

A 48-year-old man is awakened from his afternoon siesta by an agonizing pain at the base of his big toe on the right side. On examination the first metatarsal phalangeal joint is swollen, red, tender and warm to touch. Analysis of the fluid from the right knee shows: Leukocytes 68,000/mm3 Neutrophils 75% Crystal analysis negatively birefringent Gram stain negative The most likely diagnosis is A Gout B Osteoarthritis C Reactive arthritis D Rheumatoid arthritis E Septic arthritis

A. gout

25 yo male who claims to have never been acutely ill in his life presents after developing an abrupt onset of fever, HA, and painful body aches. The pt also complains of a non-productive cough and a ST. IT is the middle of January and his travel hx is unremarkable. The PE reveals a temp of 39 C, red water eyes, and small tender cervical nodes. Transient scattered rhonci and localized areas of rales are noted. A direct fluorescent antibody test was ordered stat and came back positive for the suspected etiological agents. Which is the most likely? A. influenza virus B. mycoplasma pneumoniae C. respiratory syncytial virus D. streptococcus pneumoniae E. epstein-Barr virus

A. influenza virus

a 52 yo male takes lisinopril and spironolactone for HTN. He had a high BP reading at work, so he decided to double his dose of lisinopril. He starts to feel very weak and comes to your office. A EKG done. What is a likely finding? A. peaked T waves B. U waves C. Sagging ST segments D. depressed T waves E. Q waves

A. peaked T waves

A 48-year-old woman was admitted to the hospital for hysterectomy for multiple uterine fibroids. In 1980 she had undergone a left nephrectomy for pyelolithiasis and three years later a parathyroid adenoma was discovered during investigation for hypercalcemia. During her admission for subtotal parathyroidectomy, the patient complained of recurrent epigastric pain, and investigation of this revealed severe peptic ulceration associated with a raised gastrin level of 810 pg/ml (normal less than 95 pg/ml). The patient was subsequently asymptomatic but continued with regular endocrinological outpatient review. The diagnosis of Zollinger-Ellison syndrome was made and a total gastrectomy and distal pancreatectomy were performed. During the operation the pancreatic adenoma was removed, substantiating the diagnosis of Zollinger-Ellison syndrome. The final diagnosis was a multiple endocrine neoplasia 1 (MEN-1), because gastrinomas are the most common pancreatic tumors associated with MEN-1. According to this diagnosis, which organ should be particularly controlled? A Pituitary B Liver C Lung D Muscles E Heart

A. pituitary

5 month old infant is brought in with severe cough, fast breathing, and a fever for a couple days, gradually worsening. She doesn't describe any paroxysmal cough and she states that the infant in UTD with immunizations. On exam the infant is febrile, tachypneic, dehydrated, and he has nasal flaring and wheezing. On CBC, WBCs are 6,500 mm3. Differential count shows elevated lymphocytes and decreased neutrophils. Which is the most likely causative organism? A. respiratory syncytial virus B. mycoplasma pneumoniae C. haemophilus influenzae D. parainfluenza virus E. streptococcus pneumoniae

A. respiratory syncytial virus

55 yo male with persistent cough is found to have acid-fast bacilli in a sputum sample and an isolated lesion on CXR. Where is the most likely location for his pathology? A. right apex B. right lower lobe C. right visceral pleura D. mediastinum E. right middle lober

A. right apex

62 yo AA male with a hx of T2DM, CAD, HTN and a 30 pack year presents of annual. No current sx. He's taking metformin, ASA and lisinopril/HCTZ. His bp is 150/80 mm Hg right arm sitting and his HDL cholesterol is 38 and LDL is 130. According to the ACC/AHA guidelines, what addiitoan medicaiton should be added to his regimen? A. simvastatin (zocor) B. digoxin (lanoxin) C. niacin D. losartan (cozzar) E. verapamil (covera)

A. simvastatin (zocor)

during a sports PE on a 15 year old obese male, the PA records a BP of 170/100 mm Hg bilaterally in the upper extremities while seated. The femoral pulses are weaker than the radial pulses. BP in the lower extremities is 100/75 mmHg. Which of the following is the dx? A. pheo B. coarctation of the aorta C. essential HTN D. hypertrophic cardiomyopathy E. grave's disease

B. coarctation of the aorta

patient's diastolic BP drops 10 mm Hg while moving from lying to standing position. Which is responsible for this change? A. blowing off carbon dioxide B. increased interthroacic pressure C. increased peripheral resistance D. sudden peripheral vasodilation E. venous pooling

E. venous pooling

You have been assigned to an on-call shift in the emergency department. You are seeing a 44-year-old patient with complaints of chest pain. You have obtained ECG, which indicates a 2mm ST elevation in leads I and V5. The best next step in management is: A Repeat ECG in 1 hour B Check CK-MB and Troponin I C Check CBC with differentials D Perform exercise stress testing E Perform pharmacological stress testing

B. Check CK-MB and Troponin I

A 70-year-old Caucasian male presents to the emergency department with a sudden onset of speech difficulty that started 6 hours ago. He is a smoker of 1 packet a day, hypertension, and has a family history of coronary artery disease. When you talk to him he appears very frustrated because he understands everything that you say but has great difficulty speaking back to you. You suspect that he has had a stroke and predict with certainty that the location of the problem was in the A Basilar artery B Left middle cerebral artery C Left vertebral artery D Right anterior cerebral artery E Right posterior cerebral artery

B Left middle cerebral artery

A 32-year-old male is brought to the Emergency Room by his wife. He describes a 2 hour history of severe abdominal pain. He denies any past medical or surgical history. His wife stated that, on the way to the hospital, the pain was worsened every time she went over a bump in the road. His vitals are: temperature 39.5, heart rate 120, respirations 16, blood pressure 100/50. On examination his abdomen is rigid with both voluntary and involuntary guarding. The most likely diagnosis is: A Gastroenteritis B Acute peritonitis C Uncomplicated peptic ulcer disease D Diverticulosis E Biliary colic

B Acute peritonitis

What is the most common cause of small bowel obstruction in the adult population? A. Diverticulitis B Adhesions C Gallstones D Tumor E Hernia

B Adhesions

A 40-year-old chronic alcoholic presents with cough productive of large amounts of fetid sputum. He developed the cough 2 weeks ago, and it has gradually worsened over time. He has had high intermittent pyrexia for 4 days. The vitals are as follows: Temperature 102° F, PR 108min, RR 24min, BP 140/80 mm Hg. On auscultation, there is pleural rub and diminished air entry on the right side. The chest X-ray shows a large dense opacity on the right side with a fluid level. The patient is diagnosed with lung abscess. Which of the following statements is true regarding lung abscess? A Only anerobic organisms are responsible for this condition B Aspiration is the most common cause C Sputum culture is the best investigation D Computed Tomography (CT) has no role as an investigation in this condition E Treatment is with antibiotics for 3 days

B Aspiration is the most common cause

A 64-year-old female patient complains of regurgitation of undigested food, and dysphagia. She also complains of epigastric pain, abdominal bloating and steatorrhea. Physical examination reveales calcified nodules in her finger, as well tethering of the overlying skin. One of the many serologic tests for connective tissue disorder, shows strong positivity for anti-centromere antibody test. Which of the following is the possible cause of this patient's esophagitis and finger changes? A Systemic sclerosis B CREST syndrome C Systemic lupus erythematosus D Sjogrens syndrome E Polyarthritis

B CREST syndrome

A patient comes into the emergency with an area on the skin of the leg that is red, edematous, warm, and tender to the touch. The patient also has a fever and lymphadenopathy. Which antibiotic should you use as a first line treatment for the most likely infection? A Penicillin V B Cephalexin C Imipenem D Amoxicillin E Azithromycin

B Cephalexin

A 28-year-old, female, presents with complaints of polyuria and polydipsia, which have been increasing over the past month. Initially, she attributed it to the heat, but over the past week she states the she has been taking in at least 8L per day. She denies fever, chills, or any constitutional symptoms other than profound, persistent, thirst. She denies dysuria, but has recently decreased her activities due to the frequent need to urinate. Physical exam reveals a young female in no apparent distress. Vital signs are BP 120/80; pulse 90; respiration 14; Temp 37° C (98.6° F). Laboratory studies reveal a normal Chem 7, UA SG 1.002; no cells, no bacteria. The patient is observed with water restriction for eight hours. Repeat Chem 7 showed NA 149... UA SG 1.002; no cells, no bacteria. The most likely diagnosis is A Primary polydipsia B Diabetes insipidus C SIADH D Hand-Schuller-Christian Disease E Cushing's Syndrome

B Diabetes insipidus

In large intestinal obstruction from sigmoid carcinoma, what is the immediate management of choice? A Nasogastric decompression B Emergency transverse loop colostomy C Resection and colonic anastomosis D Radiation treatment to shrink the cancer E Colonoscopy and suction decompression

B Emergency transverse loop colostomy

The most common tumor found in the hand is the: A Mucous cyst. B Ganglion. C Glomus tumor. D Pyogenic granuloma. E Fibroma.

B Ganglion.

You see a 30-year-old, African-American male construction worker in your clinic, several hours after an accident at a building site. He describes an extreme increase in pain for the last two hours. At examination, you see a soiled 10-cm long and about 1-cm deep laceration on his right forearm. The wound is surrounded by massive edema. Upon touch you can feel crepitating. Furthermore, you realize a stale, sweetish odor and blue-black discoloration. Temp. 37°C, heart rate 90/min., respiration 18/min., RR 135/85 mmHg. Allergies: none known. What is the most likely diagnosis? A Tetanus B Gas gangrene C Emphysema subcutaneum D Abscess E Sepsis

B Gas gangrene

What are the symptoms of a soft tissue sarcoma? A Pain B Gradually enlarging mass C Paralysis D Excess endocrine hormone secretion E Fever

B Gradually enlarging mass

Why is a closed loop obstruction of the intestine so critical? A It has one point of obstruction B It can rapidly progress to vascular compromise with ischemia and perforation of the intestine C It sequesters fluid D It allows food in the bowel to degrade E It allows nitrogen to accumulate

B It can rapidly progress to vascular compromise with ischemia and perforation of the intestine

Which of the following statements is true regarding juvenile rheumatoid arthritis? A There must be persistent joint swelling for at least 6 months B It is the most common chronic rheumatic disease of childhood C Laboratory tests confirms the diagnosis D The condition does not put the child at any risk for long-term complications E Acetominophen is a common first-line therapy

B It is the most common chronic rheumatic disease of childhood

A 50-year-old male comes in the ER with complaints chest pain. He also gives a history of dyspnea on exertion and intermittent dizziness. On examination, he is found to have a blood pressure of 124/86 mmHg. The apical impulse is found displaced inferiorly and laterally. A systolic thrill is palpable at the base of the heart and in the carotid vessels in the expiration phase and while leaning forward. Diagnostic studies reveal the presence of left ventricular hypertrophy on EKG and echocardiogram. Calcification of the aortic valve and reduced aortic valvular diameter is also noted on echo. The significant auscultatory finding in this patient includes paradoxical splitting of S2, presence of fourth heart sound. Which of the following murmurs would most likely be heard in this patient? A High pitched, blowing, and decrescendo diastolic murmur B Low-pitched and rough ejection systolic murmur C High pitched late crescendo-decrescendo murmur D Diastolic murmur in the tricuspid area E Holosystolic murmur

B Low-pitched and rough ejection systolic murmur

A 48 year old woman complains of an itchy and bleeding lesion on her back. It has been there for 3 years and has been enlarging in size. See the attached image. What is the most likely diagnosis? A Lyme disease B Malignant melanoma C Seborrheic keratosis D Cutaneous dermatitis E Herpes zoster

B Malignant melanoma

Which of the following is indicated in the management of polycythemia vera? A Aspirin B Phlebotomy C Blood transfusion D Plasmapheresis E Observation

B Phlebotomy

A 22-year-old man presents to his primary care practitioner with a 3-day history of sharp, localized, intermittent back pain on the right side. Patient denies any physical trauma to the area. He states the pain intensifies with deep breathing, sneezing, and coughing. He also reports he is experiencing a concurrent viral respiratory illness for which he has not sought treatment. He denies any PMH other than typical childhood illnesses. His vitals are: BP: 125/76 mm Hg, pulse: 85/min,Temp: 97.8 F. Respirations: 16/min Height: 67, Weight: 170 lbs. Lung exam reveals occasional coarse rhonchi throughout all lung fields without wheezes or rales. No increase or decrease in tactile fremitus in any lung fields, egophony is not present and diaphragmatic excursion is equal bilaterally. Localized tenderness in the right back at the level of ribs 7, 8, 9 is present with deep breaths and coughing but tenderness is not elicited with palpation of the area. Skin exam reveals no rashes or other abnormal findings. The remainder of the physical examination does not demonstrate any other abnormal findings. Which of the following is the most likely diagnosis in this patient? A Pneumonia B Pleuritis C Pneumothorax D Varicella zoster E Costochondritis

B Pleuritis

A 24-year-old intravenous drug abuser attends your practice with a four-week history of fever, dry cough, and dyspnea. He has rapid breathing. There is a diffuse interstitial pattern on his x-ray. The most likely cause is A Pulmonary emboli B Pneumocystis carinii C Kaposi's sarcoma D Legionnaire's disease E Lyme disease

B Pneumocystis carinii

A 78-year-old male presents to your office on follow-up for discussion of his cholelithiasis and dyspepsia. He has a history of intermittent biliary colic, and right upper quadrant ultrasound has identified cholelithiasis previously. However, the patient has adamantly refused cholecystectomy or any treatment for this in the past. Recently, his episodes have occurred with increased frequency, and he wonders if there is anything other than surgery that can be done. You prescribe him ursodiol (Actigall), which works by what method? A Reduces biliary secretion of cholesterol and increases the cholesterol saturation of bile B Reduces biliary secretion of cholesterol and decreases the cholesterol saturation of bile C Reduces serum cholesterol and thus reduces new stone formation D Acts as a local visceral pain modulator E Acts as a central pain modulator

B Reduces biliary secretion of cholesterol and decreases the cholesterol saturation of bile

A 56-year-old female presents in her internist's office for a follow-up visit after undergoing an upper GI endoscopy. Upon review of her chart, the internist learns she is a postmenopausal nonsmoker, who recently began hormone replacement therapy. Furthermore, she was diagnosed with gastroesophageal reflux 3 years ago, and symptoms persisted despite lifestyle modification and medication with a proton pump inhibitor. The endoscopy report describes a near-normal mucosa except for occasional findings consistent with an erosive esophagitis. A biopsy taken 7-cm above the lower esophageal sphincter (LES) shows a columnar epithelium of the specialized type. Which statement best describes this patient's condition? A Her chronic reflux has resulted in esophageal adenocarcinoma, which requires surgery and radiation B She has chronic reflux with Barrett's esophagus and she should be monitored closely C She has reflux esophagitis, but no sign of Barrett's esophagus or adenocarcinoma D She most likely has esophagitis with strictures and she should be monitored closely E She most likely has squamous cell carcinoma of the esophagus and requires surgery

B She has chronic reflux with Barrett's esophagus and she should be monitored closely

A 35-year-old female comes to your office for a rash that has been getting worse over the past few months. Since it is on her face, it is embarrassing and she wants treatment for it. You have known this patient for a few years and she has no major medical problems except for mild hypertension for which she is on atenolol. She is a non-smoker but she consumes two glasses of wine everyday with her evening meal.. She has no history of allergy. She uses makeup on her face but only occasionally; she denies using any over the counter creams to treat the rash. On exam, you note the following: a healthy looking female with a reddened skin overlying the cheeks, the nose and the forehead. The rash is patchy and seems to be clustered over the areas mentioned. On closer inspection, there is telangiectasia within the rash areas; there are also some papules, some of which seem to have a pustule on the tip. There do not seem to be comedones. The neck and upper back are spared. Which of the following constitutes the initial step in the management of rosacea? A She should discontinue atenolol B She should avoid alcohol consumption C She should have an ANA test done D She should undergo isotretinoin therapy E She should undergo pulsed dye laser therapy

B She should avoid alcohol consumption - first stop all triggers then treat

A 48 year old woman complains of a 6 month history of a slowly enlarging painless left thigh mass. See the attached image. What is the mass? A Left femoral neck fracture B Soft tissue neoplasm of the lateral thigh C Infected left hip joint D Tophi from pseudogout E Sebaceous cyst

B Soft tissue neoplasm of the lateral thigh

A 65-year-old male presents to the office with 3 days of fatigue. He has a past medical history of nephrolithiasis and hypertension. His physical exam is unremarkable. His laboratory evaluation includes serum calcium of 12 mg/dLand a PTH of 550 ulEq/ml. He undergoes operative neck exploration that identifies 3 normal parathyroid glands and no identifiable left inferior gland. After localization studies demonstrate the probable location, a re-exploration of the neck yield negative results. The likely locations of a parathyroid adenoma include A Adjacent to the right basilar pulmonary artery B Superior to the thymus C Adjacent to the inferior vena cava D Adjacent to the brachial artery E Anterior to the abdominal aorta

B Superior to the thymus

A 45-year-old male presents with increasing cough and dyspnea for several months. He has also increased serum urea, nitrogen and serum creatinine. A chest X-ray shows multiple bilateral small nodules. A transbronchial biopsy shows necrotizing granulomatosis inflammation involving small peripheral arteries. What is the most possible diagnosis in this patient? A Goodpasture's syndrome B Wegener's granulomatosis C Idiopathic pulmonary hemosiderosis D Acute bacterial pneumonia E Recurrent multiple pulmonary emboli

B Wegener's granulomatosis

Which of the following intramuscular vaccinations includes live virus that should not be administered to an immune-compromised patient? A Pneumovax B Zostavax C Influenza D TDaP E Gardisil

B Zostavax

An absolute contraindication for the use of oral contraceptives is: A a history of cancer B a history of blood clots C a current smoker D a diagnosis dysmenorrhea E over age 30

B a history of blood clots

You have a 56 y/o male patient with a sudden onset of hypothyroidism lab results after starting a new medication. What medication can cause increased levels of serum TSH, and decreased levels of serum free T4? A metoprolol B amiodarone C lisinopril D hydrocholothiazide E amlodipine

B amiodarone

A 50-year-old white male comes to the ER with ophthalmoparesis, ataxia, orthostatic hypotension and global confusion. Your diagnosis is Wernicke's encephalopathy, a neurological syndrome that is thought to be caused by a deficiency of which of the following substances? A vitamin B12 B thiamine C niacin D vitamin E E vitamin B6

B thiamine

You have a 75 y/o female patient with a history of diabetes who shares that recently she has felt a sudden need to urinate, but hasn't been able to make it to the bathroom. What type of urinary incontinence is she experiencing? A stress incontinence B urge incontinence C overflow incontinence D functional incontinence E gross total incontinence

B urge incontinence

A 15-year-old girl is brought into the emergency department by her mother with a history of ingesting an unknown number of acetaminophen tablets within the past hour. Her mother reports that she has seemed depressed recently. The patient states she failed to be accepted for the cheerleading team, came home, and took some pills she found in the medicine cabinet. When asked, she said they were the pills she took when she had a headache. According to the mother, the only pills in the medicine cabinet were acetaminophen. The acetaminophen was a large sized bottle of regular strength adult tablets. Neither the mother nor the daughter had any idea how many pills had previously been used or were ingested. The daughter stated she took a lot of pills. The mother stated her daughter had no other medical problems. Both the mother and daughter said that no other medications were being taken. On examination, her vital signs are VS T 37.5, P 78, R 18, BP 110/70, weight 55kg. She is alert and oriented. Skin is pink with good perfusion, no rashes, pupils are equal, round, reactive, oral mucosa is moist, heart is regular with a normal rhythm and rate, lungs are clear with good aeration, abdomen is soft, with normoactive bowel sounds, no palpable masses or organomegaly, no rebound, and no guarding, and neurological examination is normal. The most appropriate time to evaluate an acetaminophen level after the reported ingestion to determine if therapy is indicated is A. 8 hours B. 4 hours C. 12 hours D. 24 hours E. 2 hours

B. 4 hours

Which of the following is part of the clinical criteria in diagnosing Bacterial Vaginosis (BV)? A A vaginal fluid pH of <4.5 B A fishy odor of vaginal discharge before or after the addition of 10% KOH C Thick, cottage cheese-like vaginal discharge D Presence of budding hyphae on microscopic examination E Frothy, malodorous, yellow-green discharge

B. A fishy odor of vaginal discharge before or after the addition of 10% KOH

Early one afternoon, an adolescent boy presents with abdominal pain, nausea, and vomiting. The pain has been worsening since the onset of symptoms in the morning. There is no known gastrointestinal disease in the history or in the immediate environment. Physical examination finds no abdominal tenderness, but psoas sign and tenderness on rectal examination are detected. Temperature and pulse are slightly elevated. Skin turgor is reduced, and there is a 10 mm Hg drop in postural blood pressure. Laboratory studies find 18,000 white blood cells per microliter. Which of the following is the most appropriate diagnosis? A. Acute mesenteric lymphadenitis B. Acute appendicitis C. Salmonella gastroenteritis D. Meckel's diverticulitis E. Regional enteritis

B. Acute appendicitis - The lack of abdominal tenderness and positive rectal examination indicate inflammation of retrocecal or pelvic appendix. - Salmonella gastroenteritis would likely affect other persons in the child's environment. - Regional enteritis is associated with a prolonged history. - Acute mesenteric lymphadenitis poses a differential diagnostic challenge more frequently among children than adults. The diagnosis is impossible clinically, although the temperature tends to be higher and the pain is more diffuse. Culture of mesenteric nodes and serologic titers confirms Yersinia infection in some patients. - Meckel's diverticulitis is rare but impossible to distinguish from appendicitis. Considering the risk of perforation, limited observation, and, if unavoidable, over diagnosis are preferred in cases of suspected appendicitis.

A 40-year-old male, a chronic alcoholic, presents with cough productive of large amounts of fetid sputum. He developed the cough about 2 weeks ago, and it has gradually worsened over time. Now, the sputum is foul-smelling and copious. He has had high intermittent pyrexia for 4 days. The vitals are as follows: Temperature 102° F, PR 108min, RR 24min, BP 14080 mm Hg. On auscultation, there is pleural rub and diminished air entry on the right side. The chest X-ray shows a large dense opacity on the right side with a fluid level. The patient is diagnosed with lung abscess. Which of the following statements is true regarding lung abscess? A. Only anerobic organisms are responsible for this condition B. Aspiration is the most common cause C. Sputum culture is the best investigation D. Computed Tomography (CT) has no role as an investigation in this condition E. Treatment is with antibiotics for 3 days

B. Aspiration is the most common cause - It is always secondary, due to infection reaching the lungs from elsewhere. Some predisposing factors are alcohol abuse, seizures, coma, and stroke. - sx: cough, foul-smelling sputum, fever, and sometimes hemoptysis - dx: chest X-ray can demonstrate a cavity with fluid in it. But other associated conditions, such as empyema or pulmonary infarction, are best demonstrated on CT scan.

A 50-year-old woman has difficulty in many aspects of her daily life. She fears that others are looking down on her and often feels either criticized or rejected in social interactions. This has caused her to miss work on many occasions. On the rare occasion where she accepts social plans, she often cancels and stays home alone instead. What type of personality disorder does she most likely have? A Dependent personality disorder B Avoidant personality disorder C Narcissistic personality disorder D Borderline personality disorder E Schizoid personality disorder

B. Avoidant personality disorder - she wishes she could be social but can't - schizoid is that they're fine not being social

Which of the following drug classes is most effective in the treatment of hypertension in African American patients? A Angiotensin-converting enzyme inhibitors B Calcium channel blockers C Angiotensin receptor blockers D Beta blockers E Potassium-sparing diuretics

B. Calcium channel blockers

An 18-year-old male presents to your center with complaints of urethral discomfort, urethral discharge and dysuria. He complains of no other symptoms. His temperature is normal, and physical exam is unremarkable. He is sexually active, heterosexual, and had sex with three different women in the last 2 weeks. He has not used barrier protection. The urethral discharge is noted to be clear and slightly viscous in nature. When gram stained and examined, the discharge is found to contain a few WBCs and no organisms. This presentation is most suggestive of which of the following organisms? A Neisseria gonorrhoeae B Chlamydia trachomatis C Herpes simplex D Haemophilus ducreyi E Treponema pallidum

B. Chlamydia trachomatis

A 25-year-old, sexually active man notices that he has burning and pain while urinating. He also notices some urethral discharge. He sees you in your office for a consultation, and you order several laboratory tests. One of the tests that you order is a Gram stain and culture on a sample of the discharge. The results are negative and gonorrhea is ruled out. After reviewing all the lab results, you tell your patient that he has nongonococcal urethritis (NGU). What is the most likely cause of this patient's nongonococcal urethritis? A. Chlamydia psittaci B. Chlamydia trachomatis C. Ureaplasma urealyticum D. Trichomonas vaginalis E. Pneumocystis jiroveci

B. Chlamydia trachomatis - Common causes of nongonoccal urethritis are Chlamydia trachomatis, Trichomonas vaginalis, and Ureaplasma urealyticum.

A PA student is following up on a patient who was suspected of having hepatitis. He brings the lab results of the patient's serology to the resident. The serology patterns are as follows: Anti- HAV: negative HBsAg: positive Anti-HBs: negative Anti-HBc: IgG HBeAg: positive Anti-HBe: negative Anti- HVB: negative Which of the following is the most likely dx?: A Acute hepatitis B B Chronic hepatitis B with active viral replication C Chronic hepatitis B with low viral replication D Recovery from hepatitis B E Vaccination against hepatitis B

B. Chronic hepatitis B with active viral replication - Chronic because of IgG - active because of HbsAg and HBeAg is positive

Which of the following is not a common cause of aortic stenosis? A. Rheumatic heart disease B. Chronic intravenous drug abuse C. Congenital bicuspid aortic valve D. Monckeberg senile calcific changes E. Age greater than 60 years

B. Chronic intravenous drug abuse (1) Infants, children and adolescents (a) congenital aortic stenosis (b) congenital subvalvular aortic stenosis (c) congenital supraclavicular aortic stenosis (2) Young adults to middle age: (a) calcification and fibrosis of congenitally bicuspid aortic valve (b) rheumatic aortic stenosis (3) Middle aged to elderly (a) calcification of bicuspid valve (b) senile degenerative aortic stenosis (c) rheumatic aortic stenosis

A 56-year-old man presents with abdominal pain, indigestion, weight loss, nausea and vomiting, and gray colored stools for the past month. His past medical history is positive for alcoholism. Lab results demonstrate an elevated serum amylase and lipase, decrease trypsinogen, and positive fecal fat test. What is the diagnosis? A Acute pancreatitis B Chronic pancreatitis C Pancreatic carcinoma D Pacreatic abscess E Insulinoma

B. Chronic pancreatitis

A 68-year-old male presents with new onset of headache over the last 6 weeks. The patient describes severe unilateral headache that typically begins behind the right orbital and temporal areas. He denies vision disturbance, but mentions that during these episodes he has right-sided tearing of the eye, nasal congestion and clear rhinorrhea. He denies any other symptoms or symptoms in between episodes. The patient's medical history is remarkable for a history of 45 pack years and hyperlipidemia, which is treated. The patient is not currently experiencing symptoms, and his physical exam including full neurological exam is unremarkable. What is the patient's most likely diagnosis? A Central venous thrombosis B Cluster headache C Migraine, ocular variant D Intracranial neoplasm E Tension headache

B. Cluster headache

A 3-year-old male is brought to the Emergency Room by his parents because he woke up in the middle of the night with difficulty in breathing and a cough that sounds like a seal. On examination the child has fever, a harsh barking cough, a respiratory rate of 38/minute, and minimal stridor on agitation. On lung auscultation, there are no rales or wheezing. On cardiac auscultation, there is tachycardia. Radiological examination revealed the so-called "steeple sign." Which of the following is the most likely diagnosis? A Bronchiolitis B Croup C Epiglottitis D Foreign body aspiration E Pneumonia

B. Croup

A 15-year-old girl presents with 1 year of intermittent abdominal pain with nausea and occasional bloody diarrhea. She denies fever or weight loss, as well as any travel history. Past medical history is significant only for migraines. She takes a multivitamin. Her vital signs are within normal limits. She has mild diffuse abdominal tenderness to palpation and guaiac-positive stool. Her exam is otherwise normal. Her hemoglobin is 9.7, hematocrit is 28%, and her WBC is 12,000/uL. Which of the following findings would indicate a possible cure if a colectomy is performed? A Skip lesions, transmural involvement on colonoscopy B Crypt abscesses, superficial mucosal involvement on colonoscopy C Ileal disease D Fistulas, fissures E Mouth ulcers

B. Crypt abscesses, superficial mucosal involvement on colonoscopy - She has ulcerative colitis

For which of the following diseases would you perform the urine test, Osmolality? A Cushing syndrome B Diabetes insipidus C Pheochromocytoma D 11-hydroxylase deficiency E Diabetes mellitus

B. Diabetes insipidus

A 32-year-old female comes to you for a follow up visit. She was diagnosed with hyperthyroidism 2 weeks earlier, after presenting with tremors, heat intolerance, weight loss, and diarrhea. You prescribed propanolol for her, pending the results of her test during her last visit. She feels slightly better now. She denies any family history of thyroid disorder and has no drug allergies. She has a supportive husband. She does not smoke, does not use alcohol, or illicit drugs. Her physical examination is normal. She has been reading about management of hyperthyroidism, and she prefers the 131I treatment. What will be your most appropriate response to her request? A. It cannot be used for females of childbearing age B. Do a pregnancy test prior to starting therapy C. Tell her surgery is the best approach D. 131-I has been associated with a high recurrence rate E. 131-I has been linked to cancer of the thyroid

B. Do a pregnancy test prior to starting therapy

A 25-year-old man presents after experiencing pronounced shortness of breath while at the gym. You cannot obtain satisfying information from him during the interview and you think that he might be mentally challenged. On examination he appears short for his age and you notice webbed neck, dental malocclusion, and hypogonadism. Auscultation reveals high-pitched systolic ejection murmur maximal in the second left intercostal space with radiation to the left shoulder and ejection click that decreases with inspiration. His second heart sound is delayed and soft. The impulse of right ventricle is increased, and you palpate a thrill at second left intercostal space. What will be your best next diagnostic step? A Mental retardation assessment B Echocardiography/Doppler C Cardiac catheterization D Cardiac MRI E Karyotyping

B. Echocardiography/Doppler

A 24-year-old woman presents with 3 cm firm, painless, freely movable mass in her left breast. The mass does not change during her menstrual cycle and has grown slowly over the past year. The most likely diagnosis is which of the following? A Fibro-cystic changes B Fibroadenoma C Intraductal carcinoma D Intraductal papilloma E Fat cyst

B. Fibroadenoma

A 59 year old woman has an eczematous rash around the left nipple and areola. She denies any pain or tenderness. There has been no nipple discharge. See the attached image. How is a diagnosis made? A Clinical impression only B Full thickness skin biopsy of the areola and nipple C Mammography D Fine-needle aspirate E Mastectomy

B. Full thickness skin biopsy of the areola and nipple

A 24-year-old male presents with a large painless mass above his right clavicle. He reports no change in its current size over the last six months. He denies excessive fatigue/malaise, weight loss, fevers or chills. Physical exam reveals no hepatosplenomegaly or palpable cervical, axillary or inguinal lymphadenopathy. Excisional biopsy of the mass is performed and the pathologist reports the presence of "Reed-Sternberg cells". What is the most likely diagnosis? A Benign lymph node enlargement B Hodgkin's lymphoma C Non-Hodgkin's lymphoma D Acquired Immune Deficiency Syndrome E Metastatic disease

B. Hodgkin's lymphoma

According to the Eighth Joint National Committee (JNC 8), which of the following statements is true regarding management of hypertension? A. In those with comorbid conditions such as diabetes, pharmacological treatment for hypertension should not be initiated before age 40 B. Initial antihypertensive treatment in the nonblack population may include a thiazide diuretic or ACE/ARB inhibitor C. In the general black population, initial treatment should include a thiazide diuretic or ACE/ARB inhibitor D. If the target blood pressure is not reached within one week of initiating therapy, the initial dose should be increased E. In adults with no comorbid conditions, pharmacological treatment should be initiated when blood pressure is 130/70 mmHg or higher

B. Initial antihypertensive treatment in the nonblack population may include a thiazide diuretic or ACE/ARB inhibitor

A 30-year-old female presents to your emergency department following a gun shot wound to the right chest. Upon arrival she is awake, alert, and oriented. Her vitals are as follows: Temp 37.1, RR 35, BP 70/40, Pulse 140. A brief physical exam reveals decreased breath sounds on the right, with distended neck veins. Her trachea is slightly shifted to the left. What is your initial management of this patient? A Get a stat CXR B Insert a large needle into the right second intercostal space at the midclavicular line C Insert a chest tube into the fifth intercostal space at the midaxillary line D Get a stat CT of the chest E Insert a large needle in the right fifth intercostal space in the midaxillary line

B. Insert a large needle into the right second intercostal space at the midclavicular line

The family of viruses known as Herpesvirus includes Epstein-Barr virus, varicella-zoster, and herpes simplex. Herpes simplex virus infection usually occurs during infancy or early childhood, and causes a mild acute illness with stomatitis and slight fever. The virus travels up the axons of sensory nerves to the trigeminal ganglion supplying the mouth and related areas. One of the challenges in management of herpes simplex infection is that the virus is known to exhibit A Antigenic shift B Latency C Antigenic drift D Molecular mimicry E Gene conversion

B. Latency

A 22-year-old woman accompanied by her mother presents with a history of chronic diarrhea. She gives a history of large volumes of watery fecal output that is non-bloody and can be easily flushed; the diarrhea is painless and persists with fasting. Her mother complains that she eats less and has lost weight. She has no history of fever, flushing, wheezing, intolerance to heat/cold, or infection. Her family history and menstrual history are insignificant. On examination her weight is 130lb; height is 5 feet 5 inches; pulse is 80/min; BP is 100/74mmHg; and temperature is 98.8°F. The tongue is dry, but the rest of her physical examination is within normal limits. Based on this history, what is the most likely diagnosis? A. Carcinoid tumor B. Laxative abuse C. Celiac disease D. Hyperthyroidism E. Inflammatory diarrhea

B. Laxative abuse Large volumes of watery fecal output that is non-bloody, painless, and persists with fasting suggests secretory diarrhea that can result from use of stimulant laxatives like senna, bisacodyl, or castor oil1. The patient is young, dehydrated, and based on her symptoms, is likely taking laxatives to lose weight and get rid of unwanted calories. There are no signs and symptoms of other causes of secretory diarrhea, such as carcinoid syndrome, and the rest of the options do not have conditions that cause secretory diarrhea. Many patients might not admit the use of laxatives to lose weight. Carcinoid tumor can cause secretory diarrhea1, but it is unlikely in this patient, as she has no associated symptoms such as episodic wheezing or flushing. Hyperthyroidism results in diarrhea due to dysmotility1. However, the patient does not have any symptoms of hyperthyroidism, such as unexplained weight loss or heat intolerance. Inflammatory diarrhea can be ruled out in this patient, as there is no fever or signs of inflammation or bloody stools. Celiac disease causes diarrhea due to mucosal malabsorption. Patients present with multiple nutritional deficiencies and fatty diarrhea, which are not present in this patient, making this diagnosis unlikely1.

A 64-year-old woman presents with a 4-month history of worsening urine leakage. She has not had any previous treatment and had been managing by wearing pads. What description of her urine leakage would lead to a diagnosis of stress incontinence? A. Leakage when she has the urge to urinate and is not close to a bathroom B. Leakage with coughing, exercise, and standing up from sitting C. Leakage that is continuous and not accompanied by the urge to urinate D. Leakage with both coughing and with severe urgency to urinate E. Leakage that only occurs in situations where no bathroom is available

B. Leakage with coughing, exercise, and standing up from sitting - Stress incontinence is the result of dysfunction of the urethral sphincter, thereby allowing urine to leak with increased intra-abdominal pressure. - urge incontinence is Leakage when she has the urge to urinate and is not close to a bathroom - Leakage that is continuous and not accompanied by the urge to urinate is with neurogenic bladder or some other cause of urinary retention - Leakage with both coughing and with severe urgency to urinate is mixed incontinence - Leakage that only occurs in situations where no bathroom is available is functional incontinence (situational disorder)

A 56-year-old male presents to the ED with right-sided hemiparesis and sensory deficits in the face, hand and arm. When trying to explain why he is here, he is able to speak; however, he leaves out many words and seems frustrated. Which of the following arteries is most likely occluded? A Left Internal carotid artery B Left Middle cerebral artery, superior branch C Right Middle cerebral artery, inferior branch D Left Posterior cerebral artery E Right Posterior cerebral artery

B. Left Middle cerebral artery, superior branch

Which of the following criteria is consistent with Schizoid Personality Disorder? A. Desire for close personal relationships B. Little desire for close relationships C. Odd beliefs and magical thinking D. Paranoid thoughts E. Eccentric behavior

B. Little desire for close relationships

A 62 year old male presents with a 5 year history of epigastric pain and anorexia. Over the past 6 months he has lost 15 pounds. Upper endoscopy reveals a gastric carcinoma. Which of the following is a contraindication to the curative surgical resection of a gastric carcinoma? A. Location at the gastroesophageal junction B. Liver metastasis C. Weight loss D. Advanced age E. CT scan indicating enlarged perigastric lymph nodes

B. Liver metastasis A diet high in fresh vegetables is not associated with gastric cancer. Diets high in pickled vegetables and meat , however has been found to be associated with an increased risk of gastric carcinoma particularly in the Japanese population. Dietary nitrates may increase the risk of gastric carcinoma. The proposed mechanism involves the conversion of dietary nitrates to nitrites by enteric bacteria. Nitrites can combine with amides and amines to produce nitrosamines and nitrosamides which may be carcinogenic. The refrigeration of food prevents the initial reduction of nitrates to nitrites. Hypochlorhydric and achlorhydric states have been implicated in the genesis of carcinoma, the proposed mechanism is that the more neutral pH allows the predominance of nitrite forming-bacteria. Adenomatous polyps larger than 2 cm. may undergo malignant degeneration into carcinoma. Polyps with carcinoma confined to the tip provide strong evidence of their malignant potential. As with other tumors, the prognosis of gastric carcinoma is related to the stage of disease. The stage is dependent on the TNM classification of a particular tumor which is the tumor size, depth of penetration, extent of nodal involvement, and the presence or absence of metastasis. Lymphatic infiltration is actually associated with an improved prognosis in gastric carcinoma. Liver metastasis is a contraindication for curative surgical resection in gastric carcinoma. This is in contrast to colonic carcinoma where a solitary liver metastasis, which is felt to be surgically resectable is not a contraindication to the primary colonic surgery since prolonged survival can be obtained in that disease with resection of the liver metastasis. This is not the case in gastric carcinoma where survival is near 0% with liver metastasis. Advanced age and weight loss are not absolute contraindications to curative resection. The overall physiologic status of the patient must be evaluated to determine whether a particular patient is a surgical candidate. CT scan findings of enlarged gastric nodes may be secondary to reactive changes and should not preclude an attempt at surgical resection. GE junction tumors although technically challenging from a surgical standpoint are not contraindications for surgery. Of those who have surgery, about 50% are resected for cure and 5 year survival is between 20-30%.

A 76-year-old Caucasian female presents to your office with worsening loss of central vision for the past 6 months. Her peripheral vision remains unaffected. She has a past medical history of hypertension, diabetes, and smokes about 5 cigarettes per day. Red reflex is intact bilaterally and fundoscopic examination reveals macular depigmentation and drusen. What is the most likely diagnosis? A. Diabetic retinopathy B. Macular degeneration C. Open-angle Glaucoma D. Retinal detachment E. Cataracts

B. Macular degeneration

A 54-year-old man with proven alcoholic liver cirrhosis is referred to a hospital because he gave a medical history of the appearance of hematemesis and melena accompanied with fatigue, which appeared two days before admission to the hospital. He also gives a medical history of repeated episodes of hepatic encephalopathy during the past seven months. Physical examination reveals an enlarged liver that is palpated for three centimeters under right rib border, an enlarged spleen which is palpated for half a centimeter under the left rib border as well as a minimal quantity of ascites. His blood pressure is 110/65 mmHg, and his heart frequency is 89 beats/minute. The remaining physical findings are unremarkable. Laboratory analyses reveal anemia (3.1 million erythrocytes/mm3, hemoglobin 8.9 g/dL), thrombocytopenia (97,000 platelets/cubic millimeter), a slightly elevated AST (67 IU/l) and ALT (43 IU/l) levels. Serum protein level is 59 g/l and prothrombin time is slightly prolonged (one second above the upper limit) which is in accordance with diagnosis of liver cirrhosis. Other routine laboratory analyses reveal no abnormalities. Endoscopy reveals the presence of seven, small, esophageal varices and one of them actively bleeds. What is the best method for treatment in this patient? A. TIPS should be performed B. Perform sclerotherapy C. Varices should be treated by band ligation D. Propranolol should be prescribed E. Shunt operation should be performed

B. Perform sclerotherapy

A 17-year-old male comes to your clinic presenting with a rash. He states he noted a single oval patch several days before a more generalized rash erupted. On physical examination, you note a fawn-colored rash that forms a "Christmas tree" distribution following skin lines on the posterior trunk. This rash is most prevalent on the trunk, and the proximal upper and lower extremities. What is the most likely diagnosis? A Atopic dermatitis B Pityriasis rosea C Psoriasis D Lichen planus E Tinea corporis

B. Pityriasis rosea

A 26-year-old male patient visits the infectious disease clinic of a university hospital. The patient reports that he was diagnosed HIV-positive 4 months ago and wants to discuss potential treatment. Following a history, physical examination, and laboratory testing it is determined that the patient has no signs of HIV disease. His CD4+ T cell count is 400/mm3 and plasma viral load is 8,000 copies/mL. As part of the care for this patient a series of vaccinations are recommended. Which of the following is an acceptable vaccine battery? A Hepatitis A, DPT, Oral typhoid B Pneumococcus, DPT, meningococcus C Influenza, DPT, BCG D Oral typhoid, MMR, meningococcus E Hepatitis B, Hepatitis A, Yellow fever

B. Pneumococcus, DPT, meningococcus

A 22-year-old female presents with a 2-week history of mild fever, fatigue, weight loss, and diarrhea. Recently she reports coughing up blood. Upon clinical exam, rales were auscultated and difficulty in breathing was noted. What is the most likely diagnosis? A Cystic fibrosis B Pulmonary tuberculosis C Pulmonary histiocytosis D Pulmonary aspergilloma E Pulmonary nocardiosis

B. Pulmonary tuberculosis

A 22-year-old female presents with a 2-week history of mild fever, fatigue, weight loss, and diarrhea. Recently she reports coughing up blood. Upon clinical exam, rales were auscultated, difficulty in breathing was noted, and a positive Babinski reflex was also noted. What is the most likely diagnosis? A. Cystic fibrosis B. Pulmonary tuberculosis C. Pulmonary histiocytosis D. Pulmonary aspergilloma E. Pulmonary nocardiosis

B. Pulmonary tuberculosis

A 68-year-old woman presents with episodic, monocular blindness lasting typically less than 5 minutes described as a curtain moving vertically over her visual field. She denies pain or other related vision symptoms. Fundoscopic exam reveals no significant abnormality. This condition described above is most likely caused by A. Detached retina B. Retinal artery emboli C. Retinal vein occlusion D. Papilledema E. Macular degeneration

B. Retinal artery emboli

A 13-month-old male presents to the ED with the chief complaint of rash. The mother tells you that the child has had high fevers for the past 4 days although he hasn't had a fever for the past 24 hours. The rash began 6 hours ago and started on his chest and back and spread to his neck, face, and arms. The child doesn't appear to be itchy and has been acting normally since the fever subsided. The mother denied cough, runny nose, vomiting, and diarrhea. The only medication that the child has taken is acetaminophen. On examination the child is happy and playful. The only physical finding is the rash that is shown in the image. The condition that this child has is most likely A. Rubella B. Roseola C. Measles D. Drug hypersensitivity E. Varicella

B. Roseola - Roseola infantum (also called exanthem subitum) is caused by Human Herpesvirus-6 and occurs almost exclusively during infancy. There is usually no prodromal period. - high temperatures for 3 - 5 days - rash is rose-colored and begins as discrete, small, slightly raised pink lesions on the trunk and spreads to the neck, face and proximal extremities.

A 56-year-old man wakes up in the morning to find that he has a swollen, red, painful big toe on his left foot. Two days earlier, he had been on a cruise to the Bahamas and spent much of the time eating and drinking. He normally has one glass of wine with dinner on the weekends, but while on the cruise, his alcohol consumption increased substantially. He also did a great deal of walking in an attempt to make up for his excesses. He goes in to his physician's office, and tests are run. An x-ray shows no acute fracture, and his vital signs are within normal limits. Blood work shows an increase in uric acid but is otherwise normal. He begins treatment and feels better within next 24 hours. Which of the following is true? A Attacks are polyarticular in nature B Smaller, lower-extremity joints are usually involved first C Diet has no effect on this disease D Whites are affected more frequently than blacks E It occurs frequently in men under the age of 25

B. Smaller, lower-extremity joints are usually involved first

A thin 26-year-old female soccer player presents to your office with sudden onset of dyspnea, non-productive cough, and vague chest pain radiating to the left shoulder. Symptoms first occurred 24 hours ago, while on her usual 5-mile training run. PMH is unremarkable. EKG shows sinus tachycardia, 105bpm. Respiratory rate equals 30 breaths per minute. Decreased breath sounds and hyperresonance are noted on the left thorax; otherwise the physical exam is unremarkable. Which of the following is the most likely diagnosis? A Myocardial infarction B Spontaneous pneumothorax C Exercise-induced asthma D Dissecting aortic aneurysm E Atypical pneumonia

B. Spontaneous pneumothorax

A thin 26-year-old female soccer player presents to your office with sudden onset of dyspnea, non-productive cough, and vague chest pain radiating to the left shoulder. Symptoms first occurred 24 hours ago, while on her usual 5-mile training run. PMH is unremarkable. EKG shows sinus tachycardia, 105bpm. Respiratory rate equals 30 breaths per minute. Decreased breath sounds and hyperresonance are noted on the left thorax; otherwise the physical exam is unremarkable. Which of the following appears to be the most likely diagnosis? A. Myocardial infarction B. Spontaneous pneumothorax C. Exercise-induced asthma D. Dissecting aortic aneurysm E. Atypical pneumonia

B. Spontaneous pneumothorax - she is young, tall, thin - smoking hx is common - Initial complaints include sudden onset of chest pain, dyspnea and cough; usually associated with exertion. Chest x-rays will show air in the pleural space, along the lung border on the affected side, without evidence of pleural effusion.

A 26-year-old white woman is brought to the ER with complaints of a 2-day history of dyspnea, productive cough, and high-grade fever with chills. She has been feeling extremely fatigued for the past few days. She is a chain smoker and IV drug abuser. Vitals are as follows: HR 110/min, Temp 101 degrees F, BP 110/70 mmHg, and RR 26/min. On exam, the JVP is raised and mild hepatomegaly is present. Cardiovascular exam reveals a new pansystolic murmur at the lower left sternal border. You arrange for routine investigations, comprehensive panel, 3 sets of blood cultures, EKG, X-ray, and echocardiography. What is the most likely causative organism for the patient's condition? A Streptococcus viridans B Staphylococcus aureus C Group B streptococci D Candida albicans E Bartonella quintana

B. Staphylococcus aureus

60 yo asx male is d with small cell carcinoma of the lung based on CXR and needle biopsy. He is found to have limited stage disease. Which of the following is the best tx option for him? A. hormone modifier B. combination chemo and radiation therapy C. no tx is effective for this disease D. radiation therapy E resection of the involved lobe

B. combination chemo and radiation therapy

A 44-year-old man presents with an acute cough and 4-day history of right-sided chest pain upon inspiration. He indicates that he has been running a fever for the last few days. You ask if his cough is productive or nonproductive, and he indicates that it is productive with a yellow-green color to it. You inquire about any other symptoms, and he states that he has a history of a shaking chills when he seemed to be running a high fever. You order a chest X-ray, which indicates right middle infiltrate. You diagnose your patient with community-acquired pneumonia. What is the etiology of this case? A. Hemophilus influenza B. Streptococcus pneumoniae C. Staphyococcus aureus D. Mycoplasma pneumoniae E. Pseudomonas aeruginosa

B. Streptococcus pneumoniae The most common bacterial pathogen identified in most cases of community-acquired pneumonia is S. pneumoniae, accounting for approximately 2/3rd of bacterial isolates. Hemophilus may present similarly but is less common than S. pneumoniae. Staphyococcus may complicate pneumonia, but usually results in an empyema within the lungs. Mycoplasma is an atypical pneumonia, commonly seen in the hospital setting occurring 48 hours after admission and including at least 2 of the following symptoms: fever, cough, leukocytosis, and purulent sputum. Pseudomonas may present similar to Streptococcus but is unlikely to be seen in immunocompetent hosts.

When screening a healthy patient preoperatively for an elective operative procedure, which of the following will give the most information concerning risks for operative bleeding disorders? A Platelet count B The history C Prothrombin time D Partial thromboplastin time E Bleeding time

B. The history Explanation In an otherwise healthy individual, the history of prior surgeries or dental procedures will give the most important information about inherited coagulation disorders that would cause major bleeding problems during an operative procedure. The history should also identify patients that have been taking aspirin or nonsteroidal antiinflammatory medications since these patients will have prolonged bleeding secondary to platelet dysfunction. The absolute platelet count will be normal in these patients, but the function of the platelets will be abnormal and can be demonstrated with a bleeding time. Prothrombin time (PT) and partial thromboplastin time (PTT) are usual studies to determine the integrity of the coagulation cascade but in an otherwise healthy individual usually do not add further information in preoperative screening.

Which of the following statements is true regarding oppositional defiant disorder (ODD)? A. Oppositional behaviors must be present for at 12 months in order to meet diagnostic criteria B. The most common setting for oppositional behavior is at home C. Children with ODD have a high degree of self-awareness D. ADHD is an uncommon comorbidity E. Physical aggression is typical

B. The most common setting for oppositional behavior is at home

During a workup for secondary causes of hypertension in your 22-year-old, thin female, patient, you discovered she has fibromuscular disease (FMD) of her renal arteries. Workup also revealed a normal urinalysis, normal BUN, creatinine and a normal TSH. She takes no illicit drugs. What is the mechanism by which FMD causes hypertension? A Atherosclerotic vessels carry a reduced blood supply to the kidneys B Thickening of the renal artery causing decreased vessel diameter which eventually leads to renal artery stenosis C Decreased cardiac output and increased vascular tone lead to hypertension D Catecholamine excess from apnea causes vasoconstriction E Decreased effective circulating volume with increased renin, vasoconstriction and systemic hypertension

B. Thickening of the renal artery causing decreased vessel diameter which eventually leads to renal artery stenosis

A 2-year-old, white, male is admitted to the hospital for a blood transfusion, due to symptomatic iron deficiency anemia. PMH is unremarkable. His dietary history is significant for being a "picky eater" and for drinking more than 20 ounces of whole milk per day. He refuses to consistently swallow his oral therapy. ROS reveals a one-month history of decreased activity, lethargy, and increased drowsiness. His admission vital signs were significant for a weight of 13 kg (28.7lbs), Temp 98.7° , pulse 180, BP 100/58, respiration 20, and oxygen saturations of 94% on room air. His admission physical is significant for pallor, abnormal cardiac exam, a 5/6 SEM, and a gallop. He begins a transfusion of 100 cc's of packed red blood cells (8 cc's/kg) over 4 hours. Thirty minutes into the transfusion, you are called by nursing to evaluate and manage the patient. Upon arrival, you are provided with the following vital signs: Temp 102.5° , pulse 190, BP 85/50, respiration 25, and oxygen saturations of 94% on room air. Your patient is vomiting as you approach the bedside. What is the most likely cause of this patient's change in clinical status? A Disseminated Intravascular Coagulation (DIC) B Transfusion Reaction C Exacerbation of iron deficiency anemia D Sepsis E Celiac Disease

B. Transfusion Reaction

52 yo obese waitress presents to office complaining of left ankle and calf swelling for the past month and an occasional feeling of heaviness, aching, and cramping in her left leg, especially after being on her feet all day. PMHx shows a hx of left lower extremity DVT age 44, but no other illnesses. PE shows nontender pitting edema of left lower leg and ankle, as well as multiple superficial varicosities. Which is the most likely dx? A. AV malformation B. chronic venous insufficiency C. pre-tibial myxedema D. lmphedema E. peripheral artery disease

B. chronic venous insufficiency

A 43-year-old male presents with a 3-day history of moderate gnawing right upper quadrant pain. He has also appreciated an increase in abdominal girth going from a 30 inch waist to a 38 inch waist over several weeks time. His past medical history includes a pulmonary embolism after an inguinal hernia repair secondary to an unknown protein C deficiency. He has been on coumadin since this diagnosis. He is married and does not smoke. His physical examination includes a protuberant abdomen, mild right upper quadrant tenderness, hepatosplenomegaly, and a positive fluid wave. His laboratory analysis includes a normal white blood cell count (8300 cells/mm3), hemoglobin (13.7 g/dl), normal alkaline phosphotase (45 U/L), but severely elevated bilirubin (5.0 mg/dl), SGOT (4000 IU/L), and SGPT (4500 IU/L). A gallbladder and liver ultrasound reveal a distended liver but normal gallbladder. A CT demonstrates a thrombosed hepatic vein and hepatic necrosis. What is the best tx for this disease entity? A. Thrombolytic therapy B. Transjugular intrahepatic portosystemic shunt C. Transhepatic angioplasty D. Sodium restriction E. Surgical shunt

B. Transjugular intrahepatic portosystemic shunt

It is 4 o'clock on a Friday afternoon when the emergency department calls you to admit a patient. Seven hours ago, an 18-year-old woman broke up with her boyfriend and ingested 30 tablets of extra strength (500 mg) acetaminophen (Tylenol). Now she feels nauseous and fatigued. No signs of jaundice appreciated. Acetaminophen levels are elevated and the level is plotted on the upper line of the Rumack Matthew normogram. What is the most appropriate step to take with this patient? A. Treatment with N- acetylcysteine (Mucomyst), orally B. Treatment with N- acetylcysteine IV C. Dialysis D. Oral activated charcoal E. Observation with liver function panel every 4 hours

B. Treatment with N- acetylcysteine IV The pt should already have an IV line in at this point so PO wouldn't make sense Charcoal should be used before 4 hours LFT get every 24 hours for at least 4 days

While your associate is on vacation a 23-year-old patient of hers is seen by you. Patient's history reveals a past infection due to an STD that was treated successfully by your associate. Patient's main complaint at this time is that she has profuse vaginal discharge. Her past experience with the STD cautions her to be checked. Pelvic exam reveals a strawberry appearance on her cervix and vagina. Further investigation reveals that vaginal erythema is present. Discharge is frothy and yellowish green in color. Wet mount shows flagellated protozoa. The disease you consider to be the most likely cause is A Candida vaginitis B Trichomoniasis C Bacterial vaginosis D Atrophic vaginitis E Herpes simplex

B. Trichomoniasis

A 71-year-old woman presents with diffuse, ill-localized aching pain in the distal foot, which was initially relieved by hanging the foot off the edge of the bed. However, the pain has persisted for the last 3 months. The patient's past medical history includes diabetes mellitus, coronary artery disease, chronic renal insufficiency, and the undertaking of a coronary artery bypass grafting 3 years ago. Her physical examination includes a blood pressure of 140/70 mmHg, a pulse of 80/min with a regular cardiac rate and rhythm, normal carotid, radial pulses, and femoral pulses. No popliteal, dorsal pedis, or posterior tibial pulse is palpable. Her arterial duplex demonstrates evidence of superficial femoral and politeal artery occlusion with an ankle systolic pressure of 37 mmHg.An angiogram demonstrates patent aortoiliac, common, deep femoral artery with an occluded superficial femoral, and popliteal artery with reconstitution of the posterior tibial artery just proximal to the ankle. The patient undergoes a femoral to posterior tibial bypass utilizing ipsilateral saphenous vein and is discharged on the 5th postoperative day. Which of the following tests is the most appropriate for surveillance of the bypass graft? A. Radionuclide scan B. Ultrasound C. Magnetic resonance imaging D. Computed tomography E. Angiogram

B. Ultrasound

Which of the following is a special consideration in the patient with head and neck injuries? A. Minimizing facial scarring and deformity is the most important goal. B. Upper airway injuries are frequently associated with facial injuries. C. Airway obstruction cannot occur from blood in the throat, broken teeth, or relaxation of the posterior pharynx. D. Hoarseness of the voice and respiratory stridor are to be expected in midfacial injuries. E. Tracheostomies are never performed.

B. Upper airway injuries are frequently associated with facial injuries. Explanation Head and neck injured patients frequently may have associated injuries to the upper airway, which can compromise ventilation. The obtunded patient may have blood or broken teeth or vomitus in the posterior pharynx, which may occlude the airway, or a lax posterior pharynx from loss of consciousness may occlude the airway. Frequently, acute trauma to the anterior neck may cause laryngeal or tracheal fractures, which are manifested by hoarseness of the voice or respiratory stridor. Subcutaneous crepitus in the neck is an indicator of possible injury to the upper airway. The injured airway must be diagnosed early, and airway stabilization is usually obtained by tracheostomy.

A 57-year-old man is found to have a grade 3 systolic heart murmur. He has an enlarged spleen and small petechial hemorrhages on the arms and legs. Past medical history reveals mitral valve regurgitation with a grade 1 murmur, and it is determined that the patient is currently a candidate for surgical therapy. The patient is admitted and blood cultures are obtained. A hemolytic anaerobic gram-positive coccus is isolated. What is the most likely cause of this patient's endocarditis? A Streptococcus pneumoniae B Viridans streptococci C Abiotrophia D Pseudomonas aeruginosa E Peptococcus sp

B. Viridans streptococci

55 yo male with PMHx of stable angina started to have chest pain. Paramedics arrived, the pt was unresponsive, BP went from 153/94 mm Hg to 80/44 mm Hg in 5 minutes. pulses were chaotic and difficult to count initially and then absent after 5 minutes. ECG showed vetricular tachycardia which went to v. fib. He was unresponsive to CPR, shock, and vasopressor therapy. Which should be administered? A. adenosine B. amiodarone C. procainamide D. magnesium E. esmolol

B. amiodarone

69 yo presents to PP with complaints of dry cough and worsening SOB with exertion. 30 pack year smoker, FHx of idiopathic pulmonary fibrosis and occupation exposure positive for asbestos win workplace. Which of the following results on high resolution CT would be most consistent with idiopathic pulmonary fibrosis? A. interstitial fibrosis, thickened pleura and calcified plaques on the diaphragm or later chest wall B. bilateral patchy reticular opacities, clustered cystic airspacews, traction bronchiectasis C. bilateral diffuse reticular infiltrates with hilar and right paratracheal adenopathy D. bilateral small opacities in the upper lung fields E. focal consolidation wit evidence of loss of vascular markings

B. bilateral patchy reticular opacities, clustered cystic airspacews, traction bronchiectasis

74 yo male presents with complaints of productive cough. He has had pain upon breathing and fever over the past 2 days. PMHx include alcoholism over the past 10 years, HTN and hyperlipidemia. His temp is 102.4, resp. rate of 30, HR is 90, with dullness to percussion in the right upper lobe. Gram stain of sputum reveals encapsalated gram-neg bacilli, and CXR shows a consolidation with a cavitary lesion in the right upper lobe. What is the most liely organisms causing the pt sx? A. h. influ B. klebsiella pneumoniae C. m. cat D. listeria monocytogenes E. streptococcus. pneumo

B. klebsiella pneumoniae

50 yo male working in cotton fields, comes in with complaints of increasing SOB, fever, weight loss and night sweats for several months. Pt is a non-smoker. A CXR reveals hilar lymphadenopathy. A transbronchial biopsy is performed and microscopically demonstrates noncaseating granuloma. Which of the following is the most likely dx? A. adenocarcinoma B. sarcoidosis C. histoplasmosis D. idiopathic pulmonary fibrosis E. berylliosis

B. sarcoidosis

A 40-year-old man presents to your practice with the complaints of right upper quadrant pain, generalized weakness, and weight loss. He gives a past history of hepatitis B infection. On examination, he has hepatomegaly with arterial bruit, ascites, jaundice, and signs of cirrhosis. Which of the following investigations would you value most in the investigation of this patient? A Human chorionic gonadotrophin B α-fetoprotein C Carcinoembryonic antigen D Alkaline phosphate E S-100 antigen

B. α-fetoprotein

A 9-month-old Asian female child is seen by her pediatrician. She is suffering from failure to thrive, diarrhea, and recurrent fever. Upon examination, the physician notes that the child is pale and has an enlarged abdomen due to hepatosplenomegaly. Blood analysis reveals a decreased mean corpuscular volume. Mean corpuscular hemoglobin and hemoglobin are also decreased at 15 pg and 6 g/dl, respectively. A peripheral smear shows microcytic, nucleated red blood cells but otherwise normal morphology and a few target cells. What is the most probable diagnosis for this child? A. α - Thalassemia B. β - Thalassemia C. Hemophilia A D. Sickle cell anemia E. Von Willebrand disease

B. β - Thalassemia

in a pt with angina, which of the following EKG changes is the most indicative of acute anginal episode (particularly if the change reverses at the end of the anginal episode? A. shortened QT interval B. widened QRS complex C. prolonged PR interval D . ST segment depression E. peaked T waves

D . ST segment depression

Your patient is a 55-year-old male who presents with low back pain. His pain is exacerbated with walking,and relieved with sitting and leaning over. There is a sensation of numbness on the anterior thighs, and physical exam reveals decreased sensation with a pin-prick in the same area. Lower extremity deep tendon reflexes are 2+ bilaterally and strength is 5/5 throughout. Straight leg raise is negative bilaterally. You are awaiting MRI results, but your suspected diagnosis is what? A kyphoscoliosis B herniated lumbar disk C lumbar stenosis D peripheral neuropathy E cauda equina syndrome

C lumbar stenosis

In a patient with mixed dyslipidemia, at what triglyceride level does triglyceride-lowering become the primary goal rather than LDL-lowering? A ≥ 200 mg/dL B ≥ 150 mg/dL C ≥ 500 mg/dL D ≤ 150 mg/dL E ≥ 1000 mg/dL

C ≥ 500 mg/dL

You have an 80 y/o patient with a history of seizure disorders who described a recent event that occurred in which she was aware that she was shaking for a few seconds and the next thing she knew, she was in the ER. According to witnesses, the woman experienced a stressful event, started shaking, and then fell to the floor, losing consciousness. What type of seizure is being described? A absence seizure B myoclonic seizure C tonic-clonic seizure D complex partial seizure E simple partial seizure

C tonic-clonic seizure

A 30-year-old HIV-positive man presents to your clinic for the first time with a history of pneumocystosis and is on trimethoprim/sulfamethoxazole for secondary prophylaxis. Primary prophylaxis of Pneumocystis pneumonia is recommended for patients at which of the following CD4 cell counts? A 50 cells/mcL B 100 cells/mcL C 200 cells/mcL D 350 cells/mcL E 500 cells/mcL

C 200 cells/mcL

A 55-year old male is recovering from emergency surgery for a perforated sigmoid diverticulum. He underwent a sigmoid colectomy with a diverting colostomy five days ago. His recovery has been uneventful up until today. Yesterday he started passing flatus through the colostomy and he was placed on a clear liquid diet. His IV was heplocked this morning, and he has been ambulating in the halls throughout the day. On evening rounds, one of the nurses found him on his bed, confused and saturated with perspiration. His blood pressure was 80/50 and his heart rate is 120. You are quickly called to the bedside and examine the patient. You also note that throughout the day the patient has made only 50 cc of recorded urine output. After a 500cc. fluid challenge, the patient is still diaphoretic with a blood pressure of 84/60. The best next step in management would be: A Immediate operative intervention B Broad spectrum antibiotic coverage C Admit to ICU D One liter fluid challenge E Careful observation

C Admit to ICU

Renal stones are most often formed of what material? A Calcium phosphate B Cystine C Calcium oxalate D Uric acid E Struvite

C Calcium oxalate

A 15-year-old boy visits the emergency department complaining of a fever lasting 2 days. He has been healthy until being diagnosed with acute lymphocytic leukemia 18 months ago. Vital signs are normal with the exception of a temperature of 39° C. Physical examination is unremarkable, except for cachexia. A Gram stain of blood does not detect any organisms and laboratory results confirm neutropenia. The patient is admitted and immediately treated with intravenous ceftriaxone and tobramycin. After 24 hours, the patient remains febrile and blood cultures are negative. He complains of a sore throat and examination reveals white plaques in his pharynx and on the upper palate. A diagnosis is made and anti-infective therapy with amphotericin B is initiated. After 48 hours, blood cultures are positive and identification of the organism confirms the diagnosis. What is the most likely cause of the patient's illness? A Klebsiella pneumoniae B Staphylococcus epidermidis C Candida albicans D Pneumocystis carinii E Herpes simplex virus

C Candida albicans

Finkelstein's test (pain on grasping the thumb in the fingers and ulnar deviating the wrist) is positive in which disease? Carpal tunnel syndrome B Acute suppurative tenosynovitis C De Quervain's syndrome D Arthritis of the CMC joint of the thumb E Ulnar nerve entrapment in Guyon's canal

C De Quervain's syndrome

A 68 y/o man is admitted to the hospital with a few days' history of left lower quadrant pain with nausea and low grade fever. Examination showed that the patient had a 100.8° F fever and left lower quadrant tenderness with moderate localized rebound. The WBC count is 17,000 per cubic mm. The most likely diagnosis would be: A Internal ischemia B Carcinoma of the colon C Diverticulitis D Sigmoid valvulous E Granulomatous ileocolitis of Crohn's disease

C Diverticulitis

A deficit of which of the following neurotransmitters is most strongly associated with Parkinsonism? A GABA B Acetylcholine C Dopamine D Norepinephrine E Serotonin

C Dopamine

A patient complains that he develops shortness of breath and wheezing when he goes jogging or engages in other heavy exercise, but has no breathing difficulty otherwise. The most likely explanation for this history is which one of the following? A He is allergic to something in the environment where he jogs B He is allergic to some component of his athletic equipment C He has exercise-induced asthma D The psychological stress associated with exercise triggers latent asthma E Dyspnea and wheezing are normal responses to heavy exertion

C He has exercise-induced asthma

Pharyngitis caused by streptococcal species is common in children. IgA antibodies are an important immune response to this and other respiratory infections. The antibodies produced by the child's body as a defense against the infection may cause the most common form of vasculitis in children, which is A Stevens-Johnson syndrome B Autoimmune thrombocytopenic purpura C Henoch-Schoenlein purpura D Common variable immunodeficiency E Alder-Reilly anomaly

C Henoch-Schoenlein purpura

Why is McBurney's point an important anatomic landmark? A It is the point where abdominal aortic aneurysms rupture. B It indicates ectopic pregnancy when present. C It is the somatic site of pain when acute appendicitis has involved the parietal peritoneum. D It is the point in the right upper quadrant where an inflamed gallbladder can be palpated. E It is diagnostic of a prepyloric ulcer.

C It is the somatic site of pain when acute appendicitis has involved the parietal peritoneum.

A 75-year-old man presents to your office with complaints of dyspnea on exertion and a productive cough for the last 4 months. He also states he lost 10 pounds unintentionally in 2 months. His past medical history is significant for coronary artery disease and a myocardial infarction. He has smoked an occasional cigar the last few years. He has been retired for 12 years but for 30 years he worked odd jobs in the construction industry. He also helped his father in the family's car garage shop. Vital signs are normal. His physical exam is remarkable for decreased breath sounds in left lower lung fields and dullness to percussion. A chest radiograph is ordered and shows a left sided pleural effusion. What would you suspect in this patient? A Lung cancer B Congestive heart failure C Malignant mesothelioma D Pneumonia E Recurrent postmoyocardial infarction pericarditis

C Malignant mesothelioma

A 42 year old female presents for a one month follow-up after a medication is initiated for better management of her mixed dyslipidemia. She is also on rosuvastatin, which was initiated approximately 3 months ago. She takes both of these medications with full compliance. However, she mentions that since starting the additional medication that she her often feels like she's "on fire inside" and her skin will appear very flushed. What therapeutic agent was she prescribed at her last visit? A Ezetimibe B Fenofibrate C Niacin D Fish Oil E Cholestyramine

C Niacin

A mother brings her 2-year-old son to your office because she has concerns about his sleep. She notes that he is easy to put to sleep and has a regular bedtime at 7:30 P.M. For the past three weeks, on most nights, about 90 minutes after being put to sleep, he begins to thrash violently in the bed and, at times, lets out blood-curdling screams. He has his eyes open and seems to be talking, but he does not respond to either parent when spoken to. He sweats a lot. The episodes last about 15 minutes and then he goes back to sleep. He seems fine in the morning. He naps for about an hour in the morning and an hour in the afternoon, but these episodes do not occur with naps. The most likely diagnosis is: A Confusional arousal B Nightmares C Night Terrors D Sleep Association Disorder E Sleep Wake Transition Disorder

C Night Terrors

A 20-year-old female presents to the ER with severe pallor and purpuric spots. Physical exam is unremarkable except for the purpuric spots. Her CBC and coagulation results are as follows: RBC 3.9X106/ml Hemoglobin 9.2 mg/dl WBC 11.0x 103/ml Platelets 85x103/ml Bleeding time 11 minutes PT 14 seconds PTT 35 seconds TT 2 seconds longer than control Fibrinogen 250 mg/dl Factor VIII 100% Ristocetin cofactor assay 90% LDH 1000 U/L Haptoglobin 500mg/dl Direct Coomb's test Negative D-dimer100ng/ml Fibrin degradation products 50ug/ml Peripheral blood smear review shows marked red cell fragmentation. The most effective therapeutic approach is A Antiplatelet agents B Intravenous immunoglobulin C Plasma exchange D Platelet transfusion E Prednisone

C Plasma exchange

A 34-year-old man fractured his right mid-humerus in an automobile accident. In the emergency he notes weakness and paresthesias of his right hand. Examination reveals 2/5 weakness of the right brachioradialis, extensor carpi radialis, extensor carpi ulnaris, and extensor digitorum. Sensory exam reveals decreased light touch and pain in the dorsal aspect of the hand involving the area at the base of the thumb and first two fingers and extending to the first and second metacarpals, respectively. The brachioradialis reflex is absent. The neurologic exam is otherwise normal. This man primarily had injury to the: A Brachial plexus B Musculocutaneous nerve C Radial nerve D Ulnar nerve E Median nerve

C Radial nerve

A 65-year-old man presents to the emergency department via ambulance with signs suggestive of septic shock; the source is yet to be determined. During further workup and initial management, a physician assistant on the case tells his attending physician that he suspects the patient has cardiac tamponade due to the presence of Beck's triad. What does this triad consist of? A. Distant heart sounds, hypertension, distended neck veins B. Tachycardia, hypertension, distended neck veins C. Distant heart sounds, hypotension, distended neck veins D. Bradycardia, hypotension, bounding carotid pulses E. Tachycardia, hypotension, bounding carotid pulses

C. Distant heart sounds, hypotension, distended neck veins

A 55-year-old male presents to the Emergency Room. He has a 4-hour history of severe left-sided chest pain. His wife tells you that they went out to dinner earlier this evening, but shortly after returning home, the patient felt ill. He had several episodes of vomiting, after which the severe pain started. On exam he appears ill. He is cool and diaphoretic. Blood pressure is 90/60; heart rate is 126; respiratory rate is 32; and temperature is 39.8. There is no JVD. Cardiac exam is tachycardic, but there is no murmur. Lung sounds are decreased in the left base with dullness to percussion. The abdominal exam is tender, but there is no guarding or rebound. Laboratory studies reveal a white blood cell count of 14,000 and a hemoglobin and hematocrit of 14 and 44 gm/dl. EKG reveals a sinus tachycardia without any Q waves. Chest X-ray reveals a left pleural effusion. The described case scenario represents a patient with Boerhaave's syndrome or spontaneous esophageal rupture after forceful vomiting. Once the diagnosis is confirmed, the optimal definitive treatment consists of which one of the following? A Intravenous antibiotics B Tube thoracostomy C Surgical exploration D Observation E Pulmonary function tests

C Surgical exploration

An open wound will remain in the initial inflammatory phase until: A The wound is sterile B The critical density of ground substance is reached C The wound is covered by epithelium D The wound is covered by an eschar E Angiogenesis is complete

C The wound is covered by epithelium

The academic achievement of a previously straight A male high school student starts to fall. He also develops an unexplained tremor in his arms and dysarthria. His neurologic symptoms progress before he is finally seen by his family physician. A slit lamp examination reveals that his cornea has a distinctive rusty-brown ring. His laboratory results are as follows: Ceruloplasmin: 16 mg/dL AST (SGOT): 84 IU/L (5-40 IU/L) ALT (SGPT): 78 IU/L (5-35 IU/L) What is this disease called? A Hemochromatosis B Hemosiderosis C Wilson's disease D Whipple's disease E Wermer's syndrome

C Wilson's disease

An 80 y/o female presents with dysphagia, regurgitation, coughing after eating, aspiration, and one episode of choking which required the Heimlich maneuver 3 days ago. A modified barium swallow shows a posterior midline pouch greater than 2 cm in diameter just above the cricopharyngeus muscle. What is your diagnosis? A Esophageal stricture B Barrett's esophagus C Zenker's diverticulum D Achalasia E Esophageal cancer

C Zenker's diverticulum

A 55-year old male is recovering from emergency surgery for a perforated sigmoid diverticulum. He underwent a sigmoid colectomy with a diverting colostomy five days ago. His recovery has been uneventful up until today. Yesterday he started passing flatus through the colostomy and he was placed on a clear liquid diet. His IV was heplocked this morning, and he has been ambulating in the halls throughout the day. On evening rounds, one of the nurses found him on his bed, confused and saturated with perspiration. His blood pressure was 80/50 and his heart rate is 120. You are quickly called to the bedside and examine the patient. You also note that throughout the day the patient has made only 50 cc of recorded urine output. After a 500cc. fluid challenge, the patient is still diaphoretic with a blood pressure of 84/60. The next most logical intervention would be: A. Immediate operative intervention B. Broad spectrum antibiotic coverage C. A Swann Ganz catheter D. One liter fluid challenge E. Careful observation

C. A Swann Ganz catheter

A 63-year-old black male with a history of an inferior wall myocardial infarction at age 58 receives a 12-lead EKG as part of a work physical. His EKG is included here. His EKG shows PR intervals getting longer and loner until one QRS complex is dropped. What is this called? A AV block, 3rd degree B AV block, 1st degree C AV Block, 2nd degree, Mobitz type I D AV block, 2nd degree, Mobitz type II E Complete heart block

C. AV Block, 2nd degree, Mobitz type I

Adult bone cannot increase in length when stimulated by an excess of growth hormone because of the lack of epiphyseal cartilage, but they do increase in width by periosteal growth. In adults, an increase in growth hormone causes: A. Gigantism B. Osteomalacia C. Acromegaly D. Rickets E. Osteoporosis

C. Acromegaly - In adults, an increase in growth hormone causes *acromegaly*, a disease in which the bones, mainly the long ones, become very thick. - An excess of hormone during growing years causes *gigantism*

A 21-year-old man presents with acute onset of pleuritic chest pain accompanied by 2-3 days of fever, chills, arthralgias, and myalgias. Upon further questioning the patient notes that 4 weeks ago he had a severe sore throat and fever but was not evaluated for these symptoms. Physical examination reveals a febrile patient in mild distress. A systolic murmur is noted in the left 4th/5th intercostal space that radiates to the left axilla. A friction rub is also appreciated on exam. Laboratory results reveal an elevated erythrocyte sedimentation rate (ESR) and antistreptolysin antibodies. What is the most likely diagnosis for this patient's presentation? A Pericarditis B Pleurisy C Acute Rheumatic Fever D Endocarditis E Scarlet Fever

C. Acute Rheumatic Fever

Which of the following is true regarding routine screening mammography? A. Initial step in the eval of a breast mass B. Baseline mammorgram should be ordered at age 30 C. Can detect calcifications which are less than 1 mm in diameter D. Has a false negative rate of 2 - 4% E. Should be limited to every 2 years after 3 consecutive negative annual studies to reduce the overall radiation exposure

C. Can detect calcifications which are less than 1 mm in diameter

Immune hemolytic anemias result when antibodies are directed against a component of the erythrocyte membrane, or when drugs interact with the red cell. The cause may be a transfusion reaction or autoantibodies directed against components of the blood cell membrane. Some drugs insert into the erythrocyte membrane and antibody to the drug attaches to the cell. The test used to detect the presence of immunoglobulin on the red cell membrane is A Precipitin reaction B ELISA C Coombs test D RIA E Competitive inhibition assay

C. Coombs test

A 32 year old male is brought to the Emergency Room by his wife. He describes a 2 hour history of severe abdominal pain. He denies any past medical or surgical history. His wife stated that on the way to the hospital the pain was worsened every time she went over a bump in the road. His vitals are: temperature 39.5, heart rate 120, respirations 16, blood pressure 100/50. On examination his abdomen is rigid with both voluntary and involuntary guarding. The most likely diagnosis is acute peritonitis. Which of the following physical findings are most consistent with the presumed diagnosis? A. An abdominal mass B. Murphy's sign C. Distended abdomen with absent bowel sounds D. Visible peristalsis E. Guaiac positive stool

C. Distended abdomen with absent bowel sounds

A 60-year-old female has presented at your office for evaluation of bizarre behavior. Her daughter arrives with her but speaks with you alone and describes her mother's behavior as consisting of mood swings, lavish trips, spending foolishly, staying up at night, and being hyper. According to her daughter, her mother has been diagnosed with manic depressive illness in the past, and you feel she needs a mood stabilizer but would rather not try lithium because of the need for close monitoring. Which drug might you consider? A. Risperidone (Risperdal) B. Haloperidol (Haldol) C. Divalproex (Depakote) D. Trazodone (Desyrel) E. Trifluoperazine (Stelazine)

C. Divalproex (Depakote)

A 30-year-old male presents to the emergency department following a motor vehicle accident. The patient was a front seat passenger in a car that hit the back of another car head on at about 40 mph. His only complaint is of right foot pain. He was unable to ambulate at the scene secondary to the pain. On physical exam, his vital signs are stable. The mid-portion of his right foot is swollen and painful to touch. There is decreased range of motion on dorsal and plantar flexion of the foot. He is unable to bear weight on that foot. An X-ray of his foot reveals a fracture at the base of the second metatarsal with lateral displacement of the metatarsal base in relation to the tarsus. How would you treat a patient who presents with these findings? A. Put him in a dorsal splint and refer him to orthopedics for follow-up in the morning B. Air cast the foot and send him home on crutches for the next few days C. Elevate the leg, apply ice, and call orthopedics for a possible ORIF D. Put him in a sugar-tong splint and refer him to orthopedics for follow-up in the morning E. Apply an ace bandage to the area and send him home on crutches for the next week

C. Elevate the leg, apply ice, and call orthopedics for a possible ORIF

Which of the following statements regarding epithelial ovarian cancer is true? A. Transvaginal ultrasound is an accurate method of screening for ovarian cancer B. Most epithelial ovarian cancers occur in perimenopausal women C. Epithelial ovarian cancer is the most common ovarian cancer in the United States D. The most common type of epithelial ovarian cancer is the endometrioid type E. Epithelial ovarian cancers have the lowest mortality rate of all gynecologic malignancies

C. Epithelial ovarian cancer is the most common ovarian cancer in the United States

A 73-year-old female presents to the emergency department via EMS after a fall in her home. Her past medical history is positive for hypertension and osteoporosis; current medications include lisinopril (Zestril) and alendronate (Fosamax). She is complaining of severe left groin and thigh pain. You fear that she has a serious femoral neck fracture. What do you expect to find on physical exam? A. Internal rotation and leg shortening on the left B. Internal rotation and leg lengthening on the left C. External rotation and leg shortening on the left D. External rotation and leg lengthening on the left E. A palpable step-off fracture

C. External rotation and leg shortening on the left

A 71-year-old female presents with diffuse, ill-localized aching pain in the distal foot, which was initially relieved by hanging the foot off the edge of the bed. However, the pain has persisted for the last 3 months. The patient's past medical history includes diabetes mellitus, coronary artery disease, chronic renal insufficiency, and the undertaking of a coronary artery bypass grafting 3 years ago. Her physical examination includes a blood pressure of 140/70 mmHg, a pulse of 80/min with a regular cardiac rate and rhythm, normal carotid, radial pulses, and femoral pulses. No popliteal, dorsal pedis, or posterior tibial pulse is palpable. Her arterial duplex demonstrates evidence of superficial femoral and politeal artery occlusion with an ankle systolic pressure of 37 mmHg. An angiogram demonstrates a patent aortoiliac, common femoral artery with an occluded superficial femoral, and popliteal artery with absent distal vessels beyond the deep femoral artery that has an ostial stenosis. Which of the following is the most appropriate treatment? A. Pentoxifylline B. Profundoplasty C. Femoral to posterior tibial artery bypass D. Cilostazol E. Femoral to popliteal artery bypass

C. Femoral to posterior tibial artery bypass

A 30-year-old woman presents complaining of recurrent bilateral breast pain. She states that it seems to be worse during the last few days of her menstrual cycle. She also tells you that she has felt different size lumps in her breast that occur at the same time as the pain. She also indicates that the lumps appear to get smaller after her cycle. On examination, you note several small nodular lesions in both breasts that are freely movable. The axillary lymph nodes are unremarkable bilaterally. You suspect: A. Bowen's disease B. Phyllodes tumor C. Fibrocystic condition D. Fibroadenoma E. Fibrocarcinoma

C. Fibrocystic condition - between ages 30 - 50; pain usually worsens during the premenstrual cycle; Fluctuations in size and disappearance of masses are common with this condition - Bowen's disease is a form of squamous cell carcinoma. They typically present as well-demarcated erythematous plaques with scaling on the skin surface, typically in sun-exposed areas - Phyllodes tumor is a fibroadenoma-like tumor. These tumors can become quite large due to its rapid growth and may reoccur after excision. They commonly occur in women between the ages of 40 and 50 years. - Fibroadenoma is a common benign neoplasm in young women. The fibroadenoma is described as round, rubbery, discrete, relatively mobile, and non-tender. - Fibrocarcinomais a malignant tumor of the breast. The tumor usually consists of a nontender, firm or hard mass with poorly defined margins

A 50-year-old African-American man presents to the emergency room complaining of inability to see well for the past 2 weeks. He is a married newspaper editor and is not on any medications. He has been smoking 2 packs of cigarettes a day for the past 30 years. On examination of his right eye, there is ptosis and miosis. A chest radiograph reveals a rounded opacity in the right lung field. What is the most likely dx? A Lambert-Eaton myasthenic syndrome B Hypertrophic pulmonary osteoarthropathy C Horner's syndrome D Ectopic adrenocorticotropic hormone (ACTH) secretion E Syndrome of inappropriate antidiuretic hormone secretion (SIADH)

C. Horner's syndrome

Which of the following electrolyte imbalances occurs most frequently in patients with cancer? A Hypokalemia B Hypernatremia C Hypercalcemia D Hypomagnesemia E Hypocalcemia

C. Hypercalcemia

A 30 year old male presents concerned about a mass he's had in his right groin for the past two months. He states that the mass becomes larger with standing and coughing. Upon exam, a bulge is noted near the pubic bone on the right side. Surgical repair is indicated. What type of hernia does this patient most likely have? A Direct inguinal hernia B Ventral hernia C Indirect inguinal hernia D Umbilical hernia E Femoral hernia

C. Indirect inguinal hernia

A 28-year-old man presents for a barium enema with a 6-month history of abdominal cramping, pain, tenesmus, bloody diarrhea, and painful urgency. He also reports arthralgias but denies history of traveling abroad or use of antibiotics. On clinical exam, he has indurated and painful erythematous plaques and nodules on both shins. An abdominal exam reveals a mild tenderness in the lower left quadrant (LLQ). Stool exam is positive for occult blood and negative for ova and parasites. Lab results indicate mild leukocytosis, microcytic anemia, positive P-ANCA antibodies, and elevated ESR. What would be the most likely result from a barium enema? A Apple core appearance B Cobblestone appearance C Lead pipe appearance D String sign E Thumb printing

C. Lead pipe appearance He has ulcerative colitis

A 60-year-old male comes in with the history of sudden onset of tearing pain in the chest radiating to the back. Patient has history of poorly controlled hypertension. On examination, pulse 90/min, respiratory rate 20/min, BP 150/100 mmHg. No abnormal heart sounds are audible on auscultation. Chest X-ray reveals a widened mediastinum. You diagnose this as aortic dissection (tear in the aortic intima). Which of the following inherited condition is most likely associated with this finding? A. Turner's syndrome B. Neurofibromatosis C. Marfan syndrome D. Familial hypercholesterolemia E. Dilated cardiomyopathy

C. Marfan syndrome

A 23-year-old man presents with a 2-day history of watery nasal discharge, malaise, sneezing, and nasal congestion. On examination you notice inflammation of the nasal mucosa; pulse is 80/min; BP is 130/ 84mmHg; and temperature is 98.8°F. The rest of the examination is normal. How will you treat this patient? A Azithromycin + nasal decongestants + Ibuprofen B Ibuprofen only C Nasal decongestants + non-pharmacologic remedies like steam inhalation D Anti-viral agents + nasal decongestants + Ibuprofen E Antiviral agents only

C. Nasal decongestants + non-pharmacologic remedies like steam inhalation

The most common hormone secreted by pituitary adenomas is: A Growth hormone B Adrenocorticotrophic hormone C Prolactin D Thyroid stimulating hormone E Follicle-stimulating hormone

C. Prolactin

A 20-year-old female martial arts trainer, presents with a small reddish swelling next to a recently obtained navel piercing. She had realized the bump about two weeks ago and since then, blood has been discharged from it, recurrently. She wants it removed, since it interferes with her training and is a cosmetic issue. The lesion is about 1cm in diameter, dome-shaped, and shows increased vascularization. What is the most likely diagnosis? A. Contact dermatitis B Impetigo C Pyogenic granuloma D Cellulitis E Keloid

C. Pyogenic granuloma

A 61-year-old man with hypertension developed sudden onset of right eye blindness while shaving this morning. He describes a "curtain falling over my right eye," which cleared spontaneously after 10 minutes. He had no other concurrent neurologic symptoms and no prior episodes of similar description. His general examination reveals a blood pressure of 140/90 mm Hg, but is otherwise unremarkable. Neurologic exam is normal. Refer to the attached case. Laboratory data reveal a normal erythrocyte sedimentation rate (ESR), non-reactive VDRL, and a normal head CT scan. A carotid duplex study reveals 75% stenosis of the right internal carotid artery at the carotid bifurcation and 40% stenosis of the left internal carotid artery. What is the most appropriate management option in this patient? A. Aspirin alone at 325 mg daily B. Coumadin alone to achieve an INR between 2.5 and 3.0 C. Right carotid endarterectomy D. Right carotid endarterectomy followed by left carotid endarterectomy E. Ticlopidine alone at 250 mg twice daily

C. Right carotid endarterectomy

What is the first-line treatment for a sigmoid volvulus? A. Barium enema B. Emergency colon resection C. Rigid proctosigmoidoscopy D. Oral purgatives E. Bowel stimulants

C. Rigid proctosigmoidoscopy Explanation When sigmoid volvulus causing large bowel obstruction is suspected from the history and physical exam, or when it is seen on an abdominal radiograph, the diagnostic and therapeutic modality is rigid proctosigmoidoscopy with decompression and untwisting of the volvulus.

A 12-year-old female presents with a 1-day history of right lower quadrant pain and temperature. She complains of nausea but no vomiting. Physical examination is notable for right lower quadrant pain and tenderness with guarding. She is taken to the operating room with the presumptive diagnosis of appendicitis. Using a McBurney incision in the right lower quadrant you mobilize the cecum and find the appendix to be normal in appearance. Your next step should be: A. Close the abdomen and careful observation B. Remove the appendix and close C. Run the small bowel for any other source of the patient's symptoms D. Close the abdomen and start intravenous steroids E. Perform a right hemicolectomy

C. Run the small bowel for any other source of the patient's symptoms Even though the patient's presentation was consistent with appendicitis, finding a normal appendix should not end the search. Running the small bowel is necessary to rule out a Meckel's diverticula or evidence of terminal ileitis. Closing the abdomen and observing or starting steroids would only cause more harm. A right hemicolectomy is not indicated in this situation. The most common complication from Meckel's diverticulum is bleeding secondary to ulceration of the heterotopic gastric tissue. It is the most commonly encountered diverticulum of the small intestines and represents a true diverticulum. It is the direct result of persistence of the omphalomesenteric duct. Technetium scans can be used to identify diverticula with heterotopic gastric tissue. Acute diverticulitis although not the most common presentation can mimic acute appendicitis clinically. There is much controversy about the treatment of an incidentally found Meckel's diverticulum. The general principles are that it should be removed if there is evidence of heterotopic tissue and/or there is a narrowed neck. Other relative indications include unexplained abdominal symptoms and evidence of prior inflammation. The location on the antimesenteric border is normal and is not an indication for surgical resection. The most common location of a Meckel's is the terminal ileum usually located 45-90 cm. from the ileocecal valve.

A 5-year-old female presents to the office with multiple common warts on her fingers. The mother states that she has experienced some social embarrassment from them, and is wondering what the best treatment option for her would be. Which of the following is the best first line therapy for the patient at this time? A. No treatment B. Liquid nitrogen C. Salicylic acid D. Imiquimod E. Cantharidin

C. Salicylic acid

A 40-year-old male presents with a gradually enlarging mass in the scrotum. On examination there is a 5cm mass in the right testis. The mass measures 5x4x3 cm. The cut surface is solid, homogenous, light yellow, and contains sharply circumscribed zones of necrosis. Microscopically, the individual tumor cells are uniform, large, round to polyhedral cells. It is a distinct cell membrane with abundantly clear cytoplasm and large central nucleus with 1 or 2 prominent nucleoli. The tumor cells are characteristically arranged in nests outlined by fibrous bands and these bands are infiltrated by lymphocytes and plasma cells. The most likely diagnosis in this case is: A. Leydig cell tumor B Sertoli cell tumor C Seminoma D Teratocarcinoma E Mature teratoma

C. Seminoma

A 13-year-old boy with a 3-year history of type 1 diabetes mellitus without significant problems presents to the office for an acute visit. The patient states his morning glucose readings have risen from an average of 100 mg/dL to over 200 mg/dL over the past 2 weeks. He is currently taking 25 units of glargine (Lantus) at bedtime and 8 units of aspart (Novolog) before meals. The patient states that he has been compliant with his insulin and diet. His mother states that he recently has been having nightmares and night sweats, but he denies any other complaints. ROS and physical exam are otherwise unremarkable. This patient is most likely experiencing: A A tonic seizure B Diabetic ketoacidosis C Somogyi effect D Dawn phenomenon E The onset of puberty

C. Somogyi effect - this is when there is hypoglycemia during the late evening which causes a counterregulatory hormone response that produces *hyperglycemia* in the early morning - Dawn phenomenon is a spontaneous rise in blood glucose that occurs at the end of the night

A 61-year-old woman presents with a 3-week history of breakouts on her face. She presented with a similar complaint 6 months earlier, but did not receive treatment for it. Physical examination reveals erythema and dilated vessels on the cheeks. What would be an appropriate treatment? A. Oral Erythromycin B. Topical Isotretinoin C. Topical Metronidazole cream D. Oral sulfamethoxazole and trimethoprim E. Topical steroids

C. Topical Metronidazole cream - she has rosacea -` Oral isotretinoin is recommended for individuals with severe disease.

A 35-year old patient complains of numbness, easy fatigue, and a deep aching sensation in her left leg. Symptoms are worse during her menstrual period, and are improved when she lies down and elevates her leg. Measurement at mid calf shows a 1 centimeter difference between legs. What is the most likely diagnosis? A Lumbar nerve root irritation B Multiple sclerosis C Varicose veins D Arterial occlusion E Arterial embolism

C. Varicose veins

egophony and whispered pectoriloquy are most commonly associated with what? A. atelectasis B. pulmonary embolism C. consolidation D. destruction of the alveoli E. pneumothorax

C. consolidation

68 yo male with a 10 year hx of chronic bronchitis complains of increasing dyspnea and productive cough. PE reveals JVD, hepatomegaly, ascites, peripheral edema and a parasternal lift. EKG showed peaked P waves and right axis deviation. Echo shows normal left ventricular function. Most likely dx? A. acute inferior wall infarction B. constrictive pericarditis C. cor pulmonale D. dilated cardiomyopathy E. paraneoplastic syndrome

C. cor pulmonale

which of the following statements is the most accurate relationship between a d dimer and PE? A. if the d-dimer is elevated, the patient very likely has PE B. if the d-dimer is equivocal, the patient very likely has PE C. if the d-dimer is normal, the patient unlikely has PE D. if the d-dimer is elevated, the patient unlikely has PE E. there is no relationship between d-dimer and PE

C. if the d-dimer is normal, the patient unlikely has PE

50 yo male construction worker who renovates older buildings presents with dyspnea, cough, weight loss and chest pain over the past 2 months. He has no hx of tobacco use. A left sided pleural effusion is noted on a screening Chest film. What is the most likely dx? A. adenocarcinoma B. large cell carcinoma C. malignant mesothelioma D. small cell carcinoma E. carcinoid tumor

C. malignant mesothelioma

8 month old previously healthy infant presents to ED with a 3 day hx of clear rhinorrhea, nasal congestion, cough, and low grade fever. Parents report intermittent wheezing today but deny grunting, difficulty breathing, or apnea. On exam, there is clear nasal discharge. Oropharynx is w/o erythema or exudate, there is no nasal flaring, grounting or intercostal retractions. On auscultation, there are transmitted upper airway sounds and mild expiratory wheezes, but no crackles or rhonchi. RR is 54, pulse ox is 97% RA, rectal temp is 38.0. The best tx for this pt is: A. administration of nebulized racemic epinephrine B. intravenous administration of Azithomycin C. nasal bulb suction with saline drops and reassurance D. oral administration of amoxicillin E. 2L oxygen via nasal cannula and hospital admission

C. nasal bulb suction with saline drops and reassurance

Current guidelines indicate starting a newly diagnosed patient on insulin above which AIC level? A 6.0% B 7.0% C 8.0% D 9.0% E 10%

D 9.0%

Clinical management of depressed, breast-feeding, women has often necessitated the use of antidepressants during lactation. Serum drug levels from nursing infants have shown A All antidepressant medications result in significant serum drug levels in infants of nursing mothers B Fluoxetine is the safest SSRI for nursing mothers C No antidepressants can be safely given to nursing mothers D Amitriptyline, desipramine, and sertraline are drugs of choice for nursing women E Infant serum levels of nortriptyline, paroxetine, and sertraline are detectable at high levels when taken by a nursing mother

D Amitriptyline, desipramine, and sertraline are drugs of choice for nursing women

What structure may be injured with shoulder dislocation and must be assessed after successful reduction? A Brachial artery B Median nerve C Brachial plexus D Axillary nerve E Musculoskeletal nerve

D Axillary nerve

A 32-year-old female is referred to a hospital because of the appearance of pain under the right rib border which appeared five hours ago and was preceded by a fatty meal. She states that she has been treated during the past three days with ceftriaxone for otitis media caused by penicillin-resistant bacterial strains. She also mentions that she suffers from sideropenic anemia caused by menorrhagias. Physical examination reveals a pale anicteric patient with slight tachycardia (heart rate 92 beats/minute). Her blood pressure is 120/75 mm Hg, and there is positive Murphy's sign on physical examination of abdomen. The remaining of her general physical finding is unremarkable. Laboratory analyses reveal the presence of leukocytosis (leukocyte count 11.2 x 10/9 leukocytes/liter), sideropenic anemia (erythrocyte count 2.9x 10/6 erythocytes/mm3, hemoglobin 8.6 g/dL, serum iron 48 ug/dL, TIBC 310 ug/dL, saturation index 15.5%) as well as elevation of serum alkaline phosphatase (117 IU/l, reference values 21 to 91 IU/l). Other routine laboratory analyses reveal no abnormalities. Echo sonographic examination of abdomen reveals the presence of multiple small gallstones in distended gallbladder and one of the stones is present in the cystic duct. Other abnormalities during echo sonographic examination are not noted. Cholecystectomy is performed and transfusion of one unit of blood is administered to the patient during operation. Ceftriaxone administration is continued and postoperative course is uneventful. However, sixty hours after the operation, the patient becomes jaundiced. An elevated serum bilirubin level is present (total serum bilirubin 3.2 mg/dL, conjugated bilirubin 2.3 mg/dL) and serum alkaline phosphatase level is also elevated (125 IU/l). Refer to the case. Which of the following possible etiological factors is the most likely cause of hyperbilirubinemia in this patient? A. Blood transfusion B Performed operation C Ceftriaxone administration D Biliary duct obstruction E Sideropenic anemia

D Biliary duct obstruction

A 64-year-old woman has multiple complaints including malaise, severe unilateral headache, as well as pain and stiffness in her neck, shoulders, and back. Her appetite is poor, and she has recently lost weight. Her examining physician finds that she has an oral temperature of 100.5-degree F, her hematocrit is 11.8%, and the sedimentation rate is 104 mm/hr. The provisional diagnosis made for this patient is cranial arteritis. The most feared complication of this condition is A Thrombosis of the cranial artery B Exquisite hyperesthesia C Substantial fever D Blindness E Tongue pain

D Blindness

A 28-year-old African-American woman was recently diagnosed with ovarian carcinoma. Which of the following diagnostic examinations is used as a marker for ovarian cancer? A CA 19-9 B CEA C HER 2/neu D CA-125 levels E Beta-h CG

D CA-125 levels

A 65-year-old male with well-coontrolled diabetes mellitus type II is diagnosed with essential hypertension. What medication is best for his treatment? A Metoprolol B Amlodipine C Spironolactone D Lisinopril E Diltiazem

D Lisinopril

A 75-year-old female presents with palpitations that occur sporadically over the last month. She states that occasionally she feels lightheaded with these episodes. She has a PMH remarkable for well-controlled hypothyroidism and and artificial heart valve. Physical exam reveals an irregularly irregular rhythm. An EKG done in the office is shown. What is the recommended long-term anticoagulant therapy for this patient? A Unfractionated heparin B Pradaxa (dabigatran) C Lovenox (enoxaparin) D Coumadin (warfarin) E Eliquis (apixaban)

D Coumadin (warfarin)

One of your senior emergency medicine residents tells you that 24 hours ago he and an attending managed a critical patient who subsequently died. Today, while he was reviewing the signed, dictated chart he noted some discrepancies in what had been recorded. Some of the treatments and interventions were not dictated. The discrepancies make it look like that they didn't manage the patient well. If the chart is reviewed as is, they would both be in trouble. He'd like to correct the chart, and asks you what he should do. You advise him to A Destroy the old chart; dictate a new chart with the date and time of the incident B Dictate an addendum dated and timed for the date of the incident C Do nothing; no one will notice D Dictate an addendum, dated and timed today E Tell the attending he has a big problem

D Dictate an addendum, dated and timed today

A 44-year-old male presents to your office with fatigue, dyspnea on exertion, shortness of breath, and swelling in his legs. He states that this has been a problem for about the last 2-3 months and is worsening. He denies significant past medical history or recent illnesses, but admits that he hasn't seen a doctor in years. He denies tobacco or drug use, but admits to drinking a six-pack of beer a day for the past 20 years. You suspect a form of cardiomyopathy. What is the most likely diagnosis based on this information? A Inflammatory Cardiomyopathy B Stress Cardiomyopathy C Hypertrophic Cardiomyopathy D Dilated Cardiomyopathy E Restrictive Cardiomyopathy

D Dilated Cardiomyopathy

A 20-year-old white male comes into the clinic. His chief complaint is a 2-week history of fever, which he states started on Wednesday afternoon. The fever has been as high as 101.5° per oral thermometer, coming down with administration of Tylenol. He also has had chills, dysphagia, and anorexia. When asked, he admits fatigue. On physical exam you note he has enlarged posterior and anterior cervical lymph nodes. His past medical history is noncontributory. He states that he has been with his friends and one of them might have strep throat. His labs are all within normal range, except for the mono spot, which is positive. What is the most likely pathogen? A Streptococcus pneumoniae B Adenovirus C Cytomegalovirus D Epstein-Barr virus E Group A Streptococcus

D Epstein-Barr virus

Which of the following is the best medication to treat isolated hypertriglyceridemia? A Atorvastatin B Simvastatin C Ezetimibe D Gemfibrozil E Colesevelam

D Gemfibrozil

A 20-year-old male comes to your center after having fainted while exercising. He reports past episodic chest pain, which occurred when both resting and exercising. While taking his history, he mentions that his father passed away at age 30 after a drowning incident. On examination he is stable, PR 110, BP 125/75. There is a grade II/IV systolic ejection murmur heard on the left sternal border that does not radiate. The intensity of the murmur increases when the he bears down or stands up. The lungs are clear. The EKG show nonspecific ST and T wave changes together with left atrial enlargement. What is the most likely diagnosis? A Aortic stenosis B Restrictive cardiomyopathy C Pulmonic stenosis D Hypertrophic cardiomyopathy E Mitral stenosis

D Hypertrophic cardiomyopathy

Which of the following is true regarding molluscum contagiosum? A It is caused by a bacterial infection B It is most commonly seen in adults C The disease is not considered to be contagious D It typically presents as dome-shaped, umbilicated papules E Appropriate treatment is Mupirocin topical ointment

D It typically presents as dome-shaped, umbilicated papules

A 42-year-old male presents to the emergency department complaining of right foot pain after falling off a ladder. He lost his balance and fell approximately 9 feet, initially landing on his feet, then falling down to the left onto the ground. Examination of his right foot reveals swelling, tenderness, and ecchymosis over the posteromedial heel. His ankle has full range of motion. What is the most likely fracture given his mechanism of injury? A Calcaneal fracture B Talus fracture C Navicular fracture D Metatarsal fracture E Fibular fracture

D Metatarsal fracture

A 30 year old obese patient has been working on diet and exercise for 3 months after being diagnosed with diabetes mellitus type II. Her HbA1c level at the time of the visit is still 8.0. What is first-line pharmacological treatment at this time? A Glipizide B Glyburide C Insulin D Metformin E Gemfibrozil

D Metformin

A 5 year-old boy is noted to have rigid flat feet on examination. This is consistent with: A Talipes calcaneus B Talipes equinus C Pes cavus D Pes planus E Talipes valgus

D Pes planus

A 49 year old black female presents with worsening dyspnea and fatigue over the last 6 months. You order a chest x-ray, which is included here. What is the most likely diagnosis? A Emphysema B Lung carcinoma C Systemic Lupus Erythematous D Sarcoidosis E Amyloidosis

D Sarcoidosis

A 60-year old male has progressive paresthesia of the fifth digit of the right hand and degenerative cervical arthritis. Physical examination reveals marked restriction of neck movement and confirms decreased sensation of the small finger and the adjacent side of the ring finger along with atrophy of the hypothenar eminence. An EMG corroborates compression of the ulnar nerve at the elbow. What is the most appropriate management of this patient? A. Non-steroidal anti-inflammatory medications B Physical therapy evaluation C Pyridoxine D Surgical decompression of the cubital tunnel E CAT scan of the cervical spine

D Surgical decompression of the cubital tunnel

Which of the following is consistent with septic arthritis? A Gradual onset B Polyarticular involvement C Usually affects small joints, such as the hands and wrists D Synovial fluid white blood cell counts reveal over 80% neutrophils E Streptococcus pyogenes is the most common cause of septic arthritis in native joints

D Synovial fluid white blood cell counts reveal over 80% neutrophils

A 24 year old female presents to her primary care physician with complaints of abdominal cramps and diarrhea. The PCP refers her to gastroenterology to rule out inflammatory bowel disease. Where is the most likely location of pathology for a diagnosis of Crohn's disease? A Sigmoid colon B Proximal jejunum C Distal esophagus D Terminal ileum E Transverse colon

D Terminal ileum

A 55 y/o male presents to the ED after a fall resulting in a C6 burst fracture; it is unstable and must be operated on immediately. This patient is currently taking Coumadin for atrial fibrillation. What should be administered for rapid reversal of the medication to achieve hemostasis for surgery? A fresh frozen plasma B platelets C packed red blood cells D vitamin K E protamine sulfate

D vitamin K

A 65-year-old male presents with bilateral calf, thigh, and buttock claudication for 4 months which has progressed from 1 block to currently 20 feet of claudication distance undergoes a thoracobifemoral bypass with general anesthesia and a double-lumen endotracheal tube. A thoracotomy is performed through the 8th interspace and bilateral groin incisions are performed to expose both femoral arteries and the descending thoracic aorta. A tunnel is created extraperitoneally with a small peripheral incision in the diaphragm. Anastomosis are performed to the lower descending thoracic aorta and bilateral femoral arteries after 10,000 Units of heparin are administered. Upon completion of the procedure, protamine 200 mg is administered for heparin reversal. Post-operatively the patient becomes hypotensive 2 hours after the procedure with a systolic blood pressure of 80 mm Hg, pulse of 110/min, temperature of 37.7°C. Chest tube output is 1400cc over this 2 hour time period (700cc/hour). Hemoglobin is 7.7 mg/dl and coagulation parameters are normal including PT, PTT, platelet count, antithrombin III level, fibrin degradation products, thrombin time, fibrinogen, and bleeding time. What is the next most appropriate treatment measure? A Fresh frozen plasma B Platelets C Vitamin K D Thoracotomy E Cryoprecipitate

D Thoracotomy

A 20-year-old primigravida with 12 weeks of gestation, arrives at the antenatal clinic for a regular checkup. The patient complains of low-grade fever and swelling in her neck since 2 days. She does not have any significant medical history and denies previous blood transfusion. On questioning, she reveals that she is in a monogamous relationship with her husband along with their new pet cat. She is a homemaker and spends her free time gardening. She has received all immunizations and they are up-to-date. On examination, her vitals are temp- 99°F, PR- 88/min, BP- 110/70mmHg and RR- 20/min. She also has painless, prominent cervical lymph nodes. Abdominal examination reveals a non-tender, palpable uterus. The fetus is at increased susceptibility to which of the pathogens? A Treponema pallidum B Cytomegalovirus C Rubella D Toxoplasma gondii E Herpes simplex virus

D Toxoplasma gondii

A 50-year-old male is brought to the emergency room by the paramedics with complaints of chest pain and lightheadedness. The patient has loud heart sounds on auscultation. Arrhythmia is diagnosed on his ECG and defibrillation is performed. Refer to the image. What type of arrhythmia did the patient have before defibrillation? A Atrial fibrillation B Ventricular fibrillation C Atrial flutter D Ventricular tachycardia E Torsades de Pointes

D Ventricular tachycardia

A 42 year old white female presents to hospital for gastric bypass surgery for morbid obesity. Possible short term post-operative complications include all the following except: A Pneumonia B DVT C Anastomotic leak D Vitamin deficiency E Bleeding from anastomosis

D Vitamin deficiency

A 38-year-old female is found to have hyperparathyroidism. In addition, she is found to have pancreatic islet cell tumors. The tumor is secreting insulin. She has a positive family history for this type of condition. Her laboratory results are as follows: BUN 13 mg/dL (10-20 mg/dL) Calcium 12.1 mg/dL (8.5-10.5 mg/dL) Potassium 4.1 mEq/L (3.5-5.0 mEq/L) Sodium 139 mEq/L (135-145 mEq/L) glucose (fasting) 58 mg/dL (65-110 mg/dL) What is the most likely syndrome? A Weil's syndrome B Waterhouse-Friderichsen syndrome C Wiskott-Aldrich syndrome D Wermer's syndrome E Wolff-Parkinson-White syndrome

D Wermer's syndrome

What is the approximate frequency of sickle cell trait in the African-American population? A. 1/400 B. 1/200 C. 1/100 D. 1/10 E. 1/2

D. 1/10

A post- gastrectomy patient on post-operative day 1 is found in the early morning with temperature of 102° F orally, and the pulse rate was 112. The most likely cause is: A Wound infection B Phlebitis C Urinary tract infection D Atelectasis E Anxiety syndrome

D. Atelectasis

How should penetrating injuries to the neck be treated? A They should be bandaged and observed B The wound should be locally debrided and antibiotic ointment should be applied C Immediate surgery and exploration of the neck should be performed D Careful radiologic and endoscopic evaluation should be performed prior to any surgical exploration of the wound E Probe the wound with a cotton-swabbed stick: if the wound is not very deep, then leave it alone

D. Careful radiologic and endoscopic evaluation should be performed prior to any surgical exploration of the wound

A 42-year-old male is admitted through the ER with a history of fever with chills, cough with greenish yellow sputum, and chest pain for the last 3 days. He finds himself short of breath on walking one block. He has body aches, headache, and lack of appetite. Since this morning he has been nauseous and has thrown up 3 times, including once in the ER. His past medical history is significant for an appendectomy at age 16 and right knee arthroscopy 3 years back. He has no drug allergies. Presently he is not on any medications. Family history is significant for coronary artery disease in his father. On exam, he has a temperature of 101.1°F, pulse rate 104/min, BP 110/67 mm Hg, and SPO 2 91%. Chest X-ray shows consolidation in the right lower lobe of the lung. EKG has sinus tachycardia; troponin is 0.1, BUN 17 mg/dL, creatinine 1.2 mg/dL, electroytes normal, and urinalysis normal. Refer to case. Which regimen should you start pending blood and sputum cultures? A Ciprofloxacin B Doxycycline C Amoxicillin and azithromycin D Ceftriaxone and azithromycin E Amoxicillin-clavulanate

D. Ceftriaxone and azithromycin - best empiric tx for pneumonia

Defective articulation secondary to motor deficits of the muscles used for speech describes which of the following etiologies? A. Dyslalia B. Dysphonia C. Dysphasia D. Dysarthria E. Cerebellar dysarthria

D. Dysarthria

A 6-month-old boy is brought into the pediatric emergency department with 1 day of vomiting, lethargy, and fever. The child was born full term after an uneventful pregnancy. He has been growing and developing normally. He has no history of surgeries or serious illnesses in the past. He is taking no medication and has no known drug allergies. There is no significant family medical history. He lives with his mother and father and is an only child. On physical exam the child is in severe distress with shallow rapid breathing and cyanosis of the hands, feet, and around the lips. His temperature is 40.1 C, heart rate 195, respiratory rate of 18 per minute, and blood pressure of 80/56. He has a tense anterior fontanel, petechial rash, and splenomegaly. The most appropriate first step in management is: A Administer broad spectrum antibiotics B Obtain a blood culture C Give a fluid bolus of ringers lactate D Endotracheal intubation E Perform a lumbar puncture

D. Endotracheal intubation

A 78-year-old female inpatient presents with a fever. She is status post-colectomy for colon cancer. What physical examination findings would support a diagnosis of superficial thrombophlebitis? A Abdominal tenderness in the right upper quadrant B Positive Homans Sign in left calf C Abdominal tenderness in the left lower quadrant D Erythema and tenderness along the vein with IV insertion E Erythema and tenderness along her incision site

D. Erythema and tenderness along the vein with IV insertion

An 18-year-old male presents to your office for a screening physical exam prior to joining the freshman lacrosse team. He reports no medical complaints and does not take any medications. Physical exam is unremarkable. His immunizations are up-to-date and he denies sexual activity or smoking. Review of routine labs reveals an elevation in unconjugated bilirubin. Liver enzymes, serum electrolytes, and complete blood count are within normal limits, as is conjugated bilirubin level. Based on this information, what is the most likely diagnosis? A Alcoholic hepatitis B Crigler-Najjar syndrome C Dubin-Johnson syndrome D Gilbert's syndrome E Infectious hepatitis

D. Gilbert's syndrome

A 45-year-old white male presents to your outpatient primary care office. He requires Hepatitis B screening for his new job. His results show the following: HBsAG: positive anti-HBc: positive IgM anti-HBc: negative anti-HBs: negative What do you tell the patient about his Hepatitis B status? A He has evidence of past Hepatitis B vaccination. B He has acute Hepatitis B infection. C He is not infected, but would be susceptible to Hepatitis B if exposed. D He has chronic Hepatitis B infection. E His results are inconclusive.

D. He has chronic Hepatitis B infection.

An 18-year-old college student presents with a bright red rash on her left cheek area that has worsened since yesterday when it first appeared. It is now becoming more tender and she developed a temperature elevation of 100.2°F taken at home. She denies any new soaps or facial creams, and wears occasional make-up. She denies any ill contacts. Her current temperature is 101.2 and she feels ill. Your most likely diagnosis is erysipelas. She denies any medication allergies. The most important next step in the management of this patient is A. Blood cultures B. CBC with differential, AST, and ALT C. Penicillin VK 250 mg 4 times daily for 7 days D. Intravenous penicillin G for 48 hours, then Penicillin VK 250 mg 4 times daily for 7 days E. Intravenous Erythromycin for 48 hours then Erythromycin 250 mg orally 4 times daily for 7 days

D. Intravenous penicillin G for 48 hours, then Penicillin VK 250 mg 4 times daily for 7 days

Which of the following is the most characteristic of the clinical and radiographic features of osteoarthritis? A Bilaterally symmetrical joint swelling B Predominant involvement of the wrist joints C Periarticular erosions D Joint space narrowing with osteophytes E Elevated erythrocyte sedimentation rate

D. Joint space narrowing with osteophytes

A 27-year-old man with Marfan syndrome presents due to exercise intolerance and heart palpitations. On exam, you note a mid-systolic click and late systolic murmur heard at the apex of the heart. The click and murmur are noted later in systole with squatting and earlier in systole with sudden standing. What is the most likely dx? A Aortic stenosis B Aortic regurgitation C Mitral stenosis D Mitral valve prolapse E Aortic dissection

D. Mitral valve prolapse

A patient presents to the emergency room in acute respiratory distress. He is a 43-year-old known HIV patient who has progressed to a full-blown AIDS infection and has recently been undergoing chemotherapy with no effect on the increased viral load level. He has developed over the course of two weeks a history of night sweats, severe fatigue, chills, and fever. His physical examination was remarkable for an enlarged liver as detected by palpation 5 cm below the costal margin. The patient had an elevated body temperature of 40°C. He had a decreased WBC of 3,000/ul and a decreased Hgb of 6.8 g/dl. Electrolytes, chemistries and urinalysis were normal. Blood and stool were obtained for mycobacterial cultures. After 20 days both cultures were positive for mycobacteria that gave the following reactions: Growth rate: 20 days Nniacin nitrate reduction: neg optimum temp: 37 C Tween hydrolysis: neg Urease: neg The organism causing the infection in this patient is A. Mycobacterium kansasii B. Mycobacterium marinum C. Mycobacterium gordonae D. Mycobacterium avium-intracellulare E. Mycobacterium tuberculosis

D. Mycobacterium avium-intracellulare

A patient presents at a healthcare facility with fever, chills, malaise, headache, and numerous petechiae on the extremities and trunk. A lumbar puncture (LP) was done and the CSF examined. Gram staining revealed Gram negative coccobacilli. The organism was grown on peptone medium at 37°C in a chamber with carbon dioxide. It was found that the organism was susceptible to temperatures above or below 37°C. What organism caused infection in the patient? A Bordetella pertussis B Francisella tularensis C Haemophilus influenzae D Neisseria meningitidis E Yersinia pestis

D. Neisseria meningitidis

24 yo male has sudden onset of chest pain and dyspnea with no sig. PMHx or preceding trauma. vital signs are stable and patient is in no acute distress. CXR revelas loss of normal lung markings along right lung margins estimated to be about 10%. What is the most important initial tx? A. aggressive incentive spirometry every 2 hours B. insertion of a large bore catheter into the R 2nd intercostal space C. nebulizer tx with a bronchodilator D. O2 supplementation and observation E. prophylactic azithormycin

D. O2 supplementation and observation

A 15-year-old male presents for follow-up for acne vulgaris. He has been using benzoyl peroxide and topical retinoic acid for the past 4 months. He returns for a re-evaluation, as his acne appears to have worsened. He states his diet has not changed significantly, and his past medical history is unremarkable. His mother had significant acne as an adolescent. Physical examination reveals large papules and pustules on the forehead, cheeks, chin, and upper back. No nodulocystic cystic lesions are noted. The most appropriate next step in management is to prescribe: A Intralesional steroid injections B Deletion of chocolate from the diet C Increased washing of the skin D Oral tetracycline E Oral Isotretinoin

D. Oral tetracycline - For stage II or III acne vulgaris

A 20-year-old male presents with history of severe headache and intermittent palpitations. On examination, the patient has tachycardia and his blood pressure is found to be 260/160 mm of Hg. Further investigations reveal a mass measuring 8 cms in diameter in the left adrenal region. The mass is excised. Grossly, the mass is well demarcated and the cut surface shows bulging tan surface with vague lobulations. Residual adrenal gland is present at one end. Histologically, the tumor is composed of polygonal cells with nuclear atypia and pleomorphism and eosinophilic to amphophilic cytoplasm arranged in a zellballen pattern. Ultrastructurally, there are membrane bound dense core granules measuring 150-250 nm in diameter with some having wide eccentric halo between electronic dense core and limiting membrane with others having a narrow uniform halo. The most likely diagnosis in this case is A. Adrenal cyst B. Adrenal cortical adenoma C. Adrenal cortical carcinoma D. Pheochromocytoma E. Neuroblastoma

D. Pheochromocytoma

A patient presents with several months of generalized pruritis and the more recent onset of jaundice, darkening of the skin, easy bruising, and bone pain. Physical examination reveals jaundice, xanthlasmas, tendon xanthomas, hepatosplenomegaly, glossitis, and clubbing of the fingers. Laboratory evaluation shows hypercholesterolemia, anemia, elevated liver enzymes, hyperbilirubinemia, and a positive antimitochondrial antibody test. Radiographic studies reveal osteoporosis and vertebral compression. Which of the following is the most likely diagnosis? A Carcinoma of the bile duct B Sclerosing cholangitis C Systemic lupus erythematosis D Primary biliary cirrhosis E Budd-Chiari syndrome

D. Primary biliary cirrhosis

19 c/o racing heartbeat and SOB over the past 2 ours. PT reports a long 5 our flight. Hx of ankle fracture 3 weeks ago requiring ORIF. PE reveals a pulse ox of 92%, resp. of 30, HR of 120, but is otherwise unremarkable. Which of the following is the vest dx test for the patient's most likely dx? A. CXR B. ultrasound of lungs C. pulmonary function testing D. Spiral CT scan E. V/Q scan

D. Spiral CT scan

A 5 month pregnant woman is diagnosed with primary tuberculosis. Which is the drug you will not use in this patient? A. Isoniazid B. Rifampin C. Ethambutol D. Streptomycin E. Pyrazinamide

D. Streptomycin

A 3-month-old baby girl presents after becoming cyanotic post-feeding, and then crying inconsolably. The foster mother reports this is the 2nd episode of this type. The 1st episode was last week after the baby made a bowel movement, but it resolved quickly. She does not know anything about the birth history of the baby, but knows the birth mother received no prenatal care. Upon cardiac exam a loud systolic ejection murmur is ascertained at the left upper sternal border. An EKG reveals normal sinus rhythm and the chest X-ray reveals a "boot shaped" cardiac shadow. What is the most likely diagnosis? A. Atrial septal defect B. Coarctation of the aorta C. Pulmonary stenosis D. Tetralogy of Fallot E. Ventricular septal defect

D. Tetralogy of Fallot - more common in Down Syndrome children, fetal alcohol syndrome children and women with no prenatal care - sx: slowed growth and development, cyanosis at birth, and clubbing - In addition, at approximately 2-4 months of age "hypoxic spells" or "tet spells" are noted. They are described as prolonged crying spells with increasing cyanosis, and with crying there is a decrease in the intensity of the heart murmur. These spells usually occur after crying, feeding, or defecation. The heart murmur is usually audible at birth and described as a harsh systolic ejection murmur at the left upper sternal border. The EKG may reveal right ventricular hypertrophy or right atrial dilatation. The chest x-ray reveals a small heart size or "boot" shaped cardiac shadow. Atrial septal defect (ASD) is the cause of about 10% of congenital heart disease cases. With a small ASD the patient is often asymptomatic; however, with a large to moderate defect the associated symptoms often include exertional dyspnea, palpitations, cardiac failure, right ventricular heave, and cyanosis in infants. Coarctation of the aorta is the cause of about 6-8% of congenital heart disease. It is described as the narrowing of the aortic arch. An infant often has a normal exam at the time of discharge after birth; however, poor feeding, dyspnea, and poor weight gain often occur in the first 6 weeks of life. Congestive heart failure by 3 months of age occurs in 20-30% of infants. Pulmonary stenosis is the cause of about 8-12% of congenital heart disease cases and can present as a neonatal emergency. Mild cases are asymptomatic. Moderate to severe disease may present with exertional dyspnea, syncope, chest pain, right ventricular failure, and/or a parasternal heave. Ventricular Septal Defect (VSD) is the cause of about 20-25% of congenital heart disease cases. Most often a newborn has no murmur immediately after birth, and most develop symptoms in the first weeks to months of life. The associated symptoms often include tachypnea as the 1st sign followed by poor feeding, diaphoresis, and eventually failure to thrive

A Caucasian female presents for a routine physical. She states she is in excellent health and has an overall negative review of systems other than occasional fatigue which she attributes to a busy lifestyle. However, on physical exam, the physician assistant palpates a singlenodule in the left lobe of the thyroid. There is no thyroid enlargement. Which of the following would increase the likelihood of malignancy? A. The nodule is painful to palpation B. It is found to be a "hot" nodule upon testing C. The nodule is soft and mobile D. The patient is less than 30 years old E. The patient has a past medical history of MRIs of the head and neck

D. The patient is less than 30 years old

A 24-year-old female presents to the ED with abdominal cramping and vaginal bleeding. She states that she is 6 weeks pregnant. Upon pelvic examination, the cervical os is closed and bloody vaginal discharge is noted. Which of the following is the most likely diagnosis? A. Inevitable abortion B. Incomplete abortion C. Complete abortion D. Threatened abortion E. Missed abortion

D. Threatened abortion

A 44-year-old man diagnosed with AIDS presents to your clinic for follow-up. Today's laboratory CD4 count is 184 cells/mm3. You inquire about the patient's overall health, and he states that he is feeling fine. A. Aspirin daily B. Isoniazid C. Alpha-interferon D. Trimethoprim-sulfamethoxazole E. Norfloxacin

D. Trimethoprim-sulfamethoxazole - AIDS patient with a CD4 count of < 200 requires prophylactic treatment for the prevention of pneumocystis pneumonia (opportunistic infection)

A 16-year-old sexually active female adolescent is seen in the office for a 2-month history of amenorrhea. She denies having unprotected sex, but always relies upon her partner to use a condom. She has vomited in the early morning twice in the past week. She has also had vaginal spotting for 3 days, accompanied by cramping lower abdominal pain that became sharp. Onset of menses was at 12 years, with normal, regular periods since then. There is no history of sexually transmitted disease. Physical examination revealed normal vital signs. Slight right and left lower quadrant abdominal tenderness without guarding and rebound was present. The cervix was closed. No blood was seen in the vaginal vault. The uterus was not palpable. Serum ?-HCG: 5,200 mIU/ml. Vaginal spotting has increased, and abdominal pain has become more frequent. Repeat examination 3 days after the initial visit is unchanged. The uterus is still not palpable. Repeat serum ?-HCG is 6,800 mIU/ml. Transvaginal ultrasound failed to reveal an intrauterine pregnancy or gestational sac. What is the most likely diagnosis? A. Cervical ectopic pregnancy B. Choriocarcinoma C. Pseudocyesis D. Tubal ectopic pregnancy E. Very early intrauterine gestation

D. Tubal ectopic pregnancy - The absence of fetal pole or gestational sac in the uterus with elevated serum β-HCG indicates an ectopic pregnancy, of which a tubal ectopic is the most common. - Serum β-HCG is not adequately elevated to suggest choriocarcinoma. - Serum β-HCG is not elevated in pseudocyesis - Transvaginal ultrasound failed to reveal even a fetal pole or gestational sac in the uterus, which should be detected by 36 to 40 days.

A 3-month-old male infant presents for a well-baby check. There have been no other changes since the last visit. Upon exam, the infant is acyanotic and well appearing. Cardiovascular exam reveals a grade III/VI high-pitched, harsh pansystolic murmur heard best at the left sternal border, 4th interscostal space (ICS). A systolic thrill is palpable over the same area where the murmur is heard best. A mild left ventricular heave is evident. No additional murmurs are heard. The remainder of the exam is unremarkable. What is the most likely diagnosis? A. Atrial septal defect B. Coarctation of the aorta C. Aortic regurgitation D. Ventricular septal defect E. Patent ductus arteriosus

D. Ventricular septal defect ventricular septal defect (VSD). This represents a mild to moderate left-to-right shunt as a thrill and heave are present and the patient remains relatively asymptomatic. Atrial septal defect (ASD) is characterized by a right ventricular heave and an ejection systolic murmur heard at the pulmonic area. Coarctation of the aorta typically presents with pulse lag in the lower extremities and a blowing systolic murmur heard best in the left axilla and back. Aortic regurgitation is a diastolic murmur heard in the right upper sternal border. Patent ductus arteriosus (PDA) is typically characterized by bounding pulses and a rough, machinery murmur that is heard at the left sternal border or left infraclavicular area. It is often seen with VSD and coarctation of the aorta.

A 32-year-old female with a history of a schizoaffective disorder presents to the emergency department with an increase in auditory hallucinations and suicidal ideation. A psychiatrist is consulted, assesses the patient, and makes a recommendation for a 72-hour admission. After reviewing the patient's current medications and past therapeutic trials, the patient is started on clozapine. Due to the risk of toxicity with this medication, which of the following lab tests must be monitored in this patient? A. liver function tests (AST, ALT) B. renal function tests (BUN, creatinine) C. coagulation studies (PT/INR, aPTT) D. complete blood count (CBC) with absolute neutrophil count (ANC) E. fasting blood sugar

D. complete blood count (CBC) with absolute neutrophil count (ANC) Clozapine is a second generation anti-psychotic that can cause agranulocytosis so you need to get a CBC and ANC every week for the first 6 months.

40 yo male with 3 day hx of fever, shaking chills, productive cough of rust-colored sputum, pleuritic chest pain, and SOB for 1 day. The pt is a heavy smoker. PE revels BP 130/70 mm Hg, pulse 100 regular rate and rhythm, resp. 30, T 39.9. He is in moderate respiratory distress. He has coarse rhonci of the R middle and lower lobes. Most likely on sputum gram stain? A. gram neg. bacilli B. gram neg. diplococci C. gram positive bacilli D. gram pos diplococci E. gram positive cocci in clusters

D. gram pos diplococci (Streptococcus pneumoniae)

57 yo male presents to ED with fever and cough for the last few days. HE states it is worsening and now he's experiencing diarrhea too. On exam he is ill-appearing. His temp. is 103.2, lungs revelas scattered rhonchi and rales throughout. The reminder of exam is wnl. CXR revels patchy infiltrates in all lung fields. Sputum gram stain revels gram-negative coccobacilli. Based on the most likely dx, which of the following abx would be most appropriate? A. amoxicillin (amoxil) B. cephalexin (keflex) C. gentamycin (garamycin) D. levofloxacin (levaquin) E. penicillin G (benzathine)

D. levofloxacin (levaquin) - H. flu

48 yo with astham and CAD is being tx with enalapril (Vasotec) and HCTZ for her HTN. She complains of a dry, irritating, nagging cough since starting her on antihypertensive medications. You decide that the cough is bothersome enough to switch her medication in order to eliminate the side effects. What do you prescribe? A. hydralazine (apresoline) B. prazosin (minipress) C. diltiazem (cardizem) D. losartan (cozaar) E. propanolol (inderal)

D. losartan (cozaar)

50 yo male in ED with severe HA. His wife states that he is irritable today. EP shows BP 220/135 mm Hg and there is swelling of the optic disc bilaterally. Which of the following is tx of choice? A. furosemide (lasix) B. nisolodipine (Sular) C. nitroglycermin (nitrostat) D. nitroprusside (Nipride) E. verapamil (calan)

D. nitroprusside (Nipride) - Indicated for the immediate reduction of blood pressure in hypertensive crises

What nerve function is most likely to be first blocked by a local anesthetic? A. temp B. motor function C. deep pressure D. pain E. touch

D. pain Explanation The effect of local anesthetics on nerve conduction depends on the size of the nerve fibers. Small fibers, which carry pain and cold sensation, are most sensitive to local anesthetic action. Pain is usually the first sensation to be blocked. The large fibers are less sensitive and sensation in these fibers is blocked in the sequence of warmth, touch, and deep pressure. Motor function can be maintained even when pain is completely blocked.

25 yo in ED c/o chest pain that is worse with coughing and deep breathing. Deviation of trachea to left. Absent breath sounds over R upper and middle lobes. Dec. tactile fremitis over same areas. Most likely dx? A. atelectasis B. TB C. pleural effusion D. pneumothorax E. pulmonary embolism

D. pneumothorax

a patient with a suspected MI has ST depression in V1-V4 with no other EKG changes. Most likely location? A. anterolateral B. high lateral C. inferior D. posterior E. anterior

D. posterior

what does the designation T1 in TNM staging of lung cancer mean? A. ipsilateral peribronchial node involvement, but no distant metastases B. one metastatic lymph node found C. disease confined to thorax D. tumor less than 3 cm E. tumor localized to the visceral pleura

D. tumor less than 3 cm T0: no evidence of primary tumor T1: tumor < 3 cm T2: tumor > 3cm but < 7 cm (invades visceral pleura, main bronchus distal to carina) T3: tumor > 7 cm (invades chest wall, parietal pleura, diaphragm, phrenic nerve) T4: tumor of any size that invades the heart, great vessels, trachea, etc. N0: no regional lymph node masses N1: metastasis to ipsilateral peribronchial and/or ipsilateral hilar lymph nodes and intrapulmonary nodes N2: Metastasis in ipsilateral mediastinal and/or subcarinal lymph node(s) N3: Metastasis in contralateral mediastinal, contralateral hilar, ipsilateral or contralateral scalene, or supraclavicular lymph node(s) M0: no metastasis M1: distant metastasis

An adolescent male presents with severe testicular pain. What finding is most supportive of testicular torsion? A Duration of greater than 24 hours B Hematuria C Low positioning of testicles D Diarrhea E Absent cremasteric reflex

E Absent cremasteric reflex

A 2-year-old male presents to the ED with the chief complaint of refusing to walk. The mother tells you that the child was well until yesterday when he appeared to have right leg pain and began limping. The mother denies a history of trauma. Last night the child refused to ambulate and began having a slight fever. Today the child has been inconsolable and cries whenever he is carried. On physical exam, the temperature is 103° F. You note that the blood pressure is normal. The child is lying on his back with his left leg held straight out and his right leg held slightly flexed and externally rotated. There is full range of motion of the left hip, knee, and ankle. The child cries and refuses to move the right hip when you attempt to flex it. The right knee and ankle are normal on exam. Which of the following tests would confirm the diagnosis of septic arthritis? A Complete blood count B Blood culture C Radiographs of the hip D Erythrocyte sedimentation rate (ESR) E Arthrocentesis

E Arthrocentesis

A 50-year-old female describes several 'fainting spells' during the last few months. She loses consciousness and falls, but then regains consciousness quickly and is fine. The ESR is raised. On examination, there is a loud first heart sound with a pansystolic murmur. The echocardiography reveals a tumor in the left atrium attached by a pedicle to the interatrial septum. These findings are typical for A Aortic dissection B Myocardial infarction C Cardiomyopathy D Pericardial effusion E Atrial myxoma

E Atrial myxoma

A 2-month-old infant is brought to the emergency room for poor feeding, crying, limpness, and constipation for 48 hours. The doctor in charge asks the mother how she feeds the baby and finds out that she was given formulas regularly and mother added honey sometimes to make the formula sweeter. She said that she kept the honey in the kitchen cupboard. When the doctor examines the baby, she notices a very drowsy infant with flaccid paralysis. Which of the following is the most likely cause of this infant's symptoms? A Sepsis B Hypothyroidism C Congenital muscular dystrophy D Lactose intolerance E Botulism

E Botulism

A 4-month-old febrile female infant presents with loss of appetite, irritability, seizures, focal sensory and motor deficits, and an acute petechial rash. On physical examination, a bulging fontanelle is noted in addition to a rectal temperature is 102.8°F. What study would be most important in this child's evaluation? A CBC B Urinalysis C Chemistry panel D Serum glucose E CSF analysis

E CSF analysis

A 30 year old patient with a history of severe head trauma from an MVA 5 years ago comes in with complaints of increased thirst and increased urination. After some other tests, patient is given ADH (antidiuretic hormone) and there is a reduction in urine output. What is the most likely diagnosis? A Syndrome of inappropriate ADH B Type I diabetes mellitus C Type II diabetes mellitus D Nephrogenic diabetes insipidus E Central diabetes insipidus

E Central diabetes insipidus

A cyst that contains varied tissue types and may contain fragments of hair, bone, or cartilage is termed a: A cyst that contains varied tissue types and may contain fragments of hair, bone, or cartilage is termed a: A Pilonidal cyst B Pilomatrixoma C Hygroma D Pilar cyst E Dermoid cyst

E Dermoid cyst

How is mechanical intestinal obstruction treated? A. Laxatives B Careful observation C Intravenous stimulants D Enemas E Emergency surgery

E Emergency surgery

A 4-year-old female experiencing respiratory distress is brought to the emergency room by her mother. Past history is significant for a respiratory infection 2-weeks prior to this visit that seemingly had resolved uneventfully. The patient has a fever of 39.6°C and appears anxious and in distress. She appears to be laboring in her breathing. She has bilaterally enlarged cervical lymph nodes and an examination of her throat reveals a membranous posterior pharynx. Swabbing of the membrane produces bleeding. A review of her past immunization records was significant. Based on her immunization records and the clinical presentation the patient was diagnosed with an infection due to Corynebacterium diphtheriae and therapy was initiated while awaiting culture results. The clinical symptoms of an infection with this organism are due to the production of a potent A Exfoliative toxin B Elastase C Enterotoxin D Erythrogenic toxin E Exotoxin

E Exotoxin

Which of the following describes Charcot's triad which is indicative of cholangitis? A Fever, tachycardia, and hypotension B Jaundice, hepatomegaly, and RUQ abdominal pain C Ascites, hepatomegaly, and RUQ abdominal pain D Fever, splenomegaly and neutropenia E Fever, jaundice, and RUQ abdominal pain

E Fever, jaundice, and RUQ abdominal pain

A 25-year old African American female complains of chills and fever. Physical examination revealed a tenderness of the right upper quadrant of the abdomen. Her husband was seen and treated for penis discharge one month ago. The most likely diagnosis is: A Cholecystitis B Cholangitis C Echinococcal cyst D Entamoeba histolytica infection E Fitz -Hugh-Curtis syndrome

E Fitz -Hugh-Curtis syndrome

Autonomous production of erythropoietin is primarily associated with tumors. The most common tumors to be associated with erythrocytosis are: A Liver B Cerebellum C Uterus D Adrenal E Kidney

E Kidney

A 50-year-old female with a 25-year history of alcohol abuse was in the intensive care unit dependent on a ventilator. She was initially brought in 5 days previous to the ER unresponsive, hypotensive, and with a gastrointestinal bleed. She had developed fevers on her 2nd day and was placed on broad-spectrum antibiotics. She was still spiking fevers and a tracheal aspirate was obtained for Gram stain and culture. The Gram stain was significant for >25 WBC/hpf and many gram-negative rods. She had a chest radiograph performed that was significant for an infiltrate and changes consistent with multiple small abscesses. The culture was significant for a gram-negative bacillus that was a lactose fermenter; mucoid; and VP positive. The organism is a known cause of nosocomial infections and strains are known to produce extended spectrum B-lactamases (ESBLs). The organism most likely causing this nosocomial infection is A Streptococcus pneumoniae B Pseudomonas aeruginosa C Staphylococcus aureus D Enterococcus E Klebsiella pneumoniae

E Klebsiella pneumoniae

A 4-month-old male is brought to the emergency room with a temperature of 39.5°C, vomiting, and irritability. Onset has been rapid, with initial symptoms beginning within 24 hours. Physical examination reveals an irritable child with a body temperature of 37°C; heart rate of 180/min; and normal blood pressure. Meningeal irritation is present. Fontanelles are bulging. A CBC was remarkable for a WBC of 10,000/ul. Electrolytes were within normal limits. A lumbar puncture was performed and the results are shown in the table. WBC count: 150/ul WBC differential: 85% neutrophils, 15% mononuclear Glucose: 30 mg/dl Protein: 60 mg/dl Gram stain revealed gram-negative diplococci. CSF culture is pending. This patient has meningitis due to: A Mycobacterium tuberculosis B Cryptococcus neoformans C Streptococcus pneumoniae D Coxsackievirus B E Neisseria meningitidis

E Neisseria meningitidis

What condition involves trapping of the foreskin behind the glans of the penis? A Balanitis B Phimosis C Prostatitis D Priapism E Paraphimosis

E Paraphimosis

Which of the following is a common complication of untreated mitral valve regurgitation? A Ventricular tachycardia B Endocarditis C Diastolic dysfunction D Increased ejection fraction E Pulmonary hypertension

E Pulmonary hypertension

A 47-year-old Latino male presents to your office complaining of worsening stomach pain. He has tried over the counter antacid and H2-blocker therapy with some relief, but now is taking them on a daily basis, and his symptoms persist. He describes his pain as being in the mid-epigastrium. Lately, he notes that he feels full about halfway through his meals, and he thinks that he might have lost some weight over the past few months from eating less, due to this discomfort. Examination reveals mild discomfort on deep palpation of the epigastric region. What should you do next? A Begin nizatidine (Axid) 150 mg twice a day B Begin lansoprazole (Prevacid) 30 mg a day C Begin hyoscyamine (Levsin) 0.125 mg three times a day as needed for stomach pain D Check serum for antibodies to Helicobacter pylori E Refer for upper endoscopy

E Refer for upper endoscopy

A 50 year old patient who recently underwent an abdominal surgery is having crampy abdominal pain, vomiting, and diarrhea. Physical exam reveals abdominal distention and hyperactive bowel sounds. What do you suspect? A Diverticulitis B Ileus C Volvulus D Intussusception E Small bowel obstruction

E Small bowel obstruction

A 23-year-old male presents with a chief complaint of drowsiness, headache, impaired vision, right sided weakness, and confusion over the past 24 hours. Past medical history includes being hit by a baseball in the head. Physical exam shows decreased sensation and strength in the right side, unilateral decreased sweating on the forehead, difficulty speaking, mild pupil dilation, and skull tenderness. What is the most likely diagnosis? A Epidural abscess B Cerebral abscess C Spinal cord abscess D Lobar intracerebral hemorrhage E Subdural hematoma

E Subdural hematoma

A 9-year-old girl had a fever for 4 days, and now has developed erythema of the cheeks and subsequently an erythematous maculopapular rash started spreading from her arms to her trunk and legs. She complains of itching. Her medical history is non-contributory and the results of physical examination are otherwise unremarkable. What is the appropriate therapy? A Tetracycline B Acyclovir C Penicillin V D Chloramphenicol E Supportive antipruritics

E Supportive antipruritics

A 50-year-old African American female presents with 4 weeks of lateral hip pain. She states that it occurs bilaterally but is worse on the left. It is worsened by walking and climbing steps and is alleviated by ibuprofen. While she states that she hasn't had any limitation of movement, she does mention that she just began to walk several times a week in an effort to lose weight, but has had to decrease the frequency and duration of her workouts secondary to pain. Upon exam, she has a clearly antalgic gait. There is no joint swelling or heat in the extremities. Lower extremity and thoracolumbar active range of motion is full throughout. Strength is 5/5 throughout the lower extremities. There is pain with passive external rotation of the hip and point tenderness over the greater trochanter bilaterally. There is mild crepitus but no effusion of the knees bilaterally. A Sacroiliitis B Osteoarthritis C Iliotibial band syndrome D Piriformis syndrome E Trochanteric bursitis

E Trochanteric bursitis

A 63-year-old female with history of hypertension presents to the Emergency Department complaining of severe elbow pain. She describes an acute onset of pain in her elbow, denying any known trauma to the area. The pain awoke her from her sleep last night and has worsened over the day. Current medications include hydrochlorothiazide/triamterene 25/37.5 mg once a day; this is a recent change after she developed a cough on her previous regimen of benazepril. Examination reveals a swollen, red, tender left elbow. What is the most likely diagnosis? A. Acute septic arthritis B. Acute rheumatic fever C. Palindromic rheumatism D. Acute gout E. Acute pseudogout

E. Acute gout - gout is a side effect of hydrochlorothiazide/triamterene

which of the following is the best initial tx for an HIV positive patient in whom you suspect PJP pneumonia? A. augmentin B. azithromycin C. ciprofloaxacin D. penicillin E. Bactrim

E. Bactrim

A 31-year-old, G1P0, at 35 weeks gestation, presents to the Emergency Department complaining of abdominal pain and fever. Her pain began yesterday afternoon and she has noticed some increase in her uterine contractions. She denies spontaneous rupture of membranes, vomiting, or diarrhea. On examination, her temperature is 38.2 ° C, (100.8° F) pulse 98, respiratory rate 22, and blood pressure 118/70. Tocometric tracing reveals mild uterine irritability and the fetal heart tracing has a baseline of 150's with good variability. Her uterus is tender and there is no rebound tenderness. The cervix is long and closed. Laboratory evaluation reveals liver function tests that are within normal limits, hemoglobin of 12.9g/dl, white blood cell count of 18,300/mm3, and platelet count of 250,000/mm3. Nitrazine testing of the vaginal fluid reveals a pH in the blue range and there is positive "ferning" on microscopic exam. Which of the following is the most important next step, in the management of the patient's symptoms? A Drawing a C-reactive protein level B Amniocentesis C Oxytocin induction D Drawing fetal fibronectin level E Beginning empiric antibiotics

E. Beginning empiric antibiotics

A 9-year-old girl is brought to your clinic with a complaint of a sore throat. The mother states that she began to run a fever a few days ago and complained that her throat hurt. On physical exam, you note a red throat, a red, beefy tongue, tonsillar exudates, and swollen anterior cervical lymph nodes. You order a rapid strep test, which comes back positive. It is noted in the patient's records that she is had a severe anaphylactic reaction to penicillin. What antibiotic would treat this infection while minimizing risk of invoking allergic reaction? A Augmentin B Cephalexin (Keflex) C Ciprofloxacin D Mupirocin (Bactroban) E Clindamycin

E. Clindamycin

A 75-year-old emeritus professor of philosophy faints in the midst of her lecture. Emergency 911 is called. Emergency medical services (EMS) determines that her pulse is 35 beats per minute and regular. She is transported her to your emergency room. Which of the following do you expect to see on her electrocardiogram (ECG)? A Normal sinus rhythm B First-degree heart block C Second-degree heart block type I D Atrial fibrillation E Complete heart block

E. Complete heart block

A 58-year-old man presents for a routine exam. His blood pressure is 150/96 mm Hg. He quit smoking 20 years ago, at which point he had been smoking 1 pack/day for 10 years. Family history is significant for diabetes and hypertension in his 2 elder siblings. Past medical history is insignificant, except for a fractured right radius 15 years ago and an appendectomy 30 years ago. Repeat blood pressure recordings show a BP of 148/98 mm Hg and then of 150/98 mm Hg. Fasting blood glucose is 122 mgdL. Renal and lipid profile and fundoscopy are ordered. Based on this patient's profile, what would be the best drug to start him on? A. Nifedepine B. Prazosin C. Atenolol D. Hydrochlorthiazide E. Enalapril

E. Enalapril - Based on the family history of diabetes and this patient's current hyperglycemia, Enalapril is the best choice, not HCTZ - nifedepine is a calcium channel blocker that can decrease blood pressure - Prazosin is an alpha-blocker that acts on the alpha-receptors of vascular smooth muscle. It may produce orthostatic hypotension and syncope as side effects. - Propranolol and atenolol are beta-blockers used to treat hypertension. These drugs slow the heartbeat by blocking the effect of nerve impulses to the heart and blood vessels.

A 41-year-old male is brought to the ED with acute hematemesis. Abdominal exam reveals distension, hepatosplenomegaly, and dull fluid wave, but no rebound tenderness and guarding. Which of the following is the most likely source of the upper gastrointestinal bleeding (UGI)? A Mallory-Weiss tear B Perforated duodenal ulcer C Chronic gastritis D Arteriovenous malformation (AVM) E Esophageal varices

E. Esophageal varices

on repeat exam (times 4 visits) the BP of an obese caucasian 70 yo female with a hx of diabetes mellitus is determined to be 170/85 mm Hg R arm sitting. She has no evidence of retinopathy, cardiac or renal abnormalities. According to the JNC 8 guidelines, what is the tx? A. Jabetolol (normodyne) and lisinopril (zestril) B. losartan (cozaar) and lisinopril (zestril) C. HCTZ (hydrodiuri) D. atenolol (tenormin) E. HCTZ (diuril) and enalapril (vasotec)

E. HCTZ (diuril) and enalapril (vasotec)

What physical exam finding(s) would support a diagnosis of phimosis? A Entrapment of the foreskin behind the glans penis in an uncircumcised male B Erythema and edema of the glans penis in an uncircumcised male C Erythema and edema of the phallus in a circumcised male D Erythema and edema of the glans penis in a circumcised male E Inability to retract the foreskin over the glans penis in an uncircumcised male

E. Inability to retract the foreskin over the glans penis in an uncircumcised male

A 35-year-old Asian man presents with diarrhea, which has worsened over the past year since he got married and his diet changed. This is associated with abdominal discomfort and the feeling of a lot of gas in his stomach. On examination, his abdomen is distended with a tympanic note on percussion. There is no area of tenderness. The hydrogen breath test reveals elevated hydrogen in his expired air. What is the most likely diagnosis? A. Celiac disease B. Whipple's disease C. Short bowel syndrome D. Tropical sprue E. Lactose intolerance

E. Lactose intolerance When the hydrogen breath test is administered, there is elevated hydrogen content in the expired air as the colonic flora digests the unabsorbed lactose.

Which of the following urine colors is found in pyuria? A. Colorless B. Yellow C. Amber D. Red-brown E. Milky white

E. Milky white

A 55-year-old Latino male, presents to the Emergency Department complaining of gross hematuria. He has been experiencing right flank pain for a few weeks, but he attributed this to muscular strain from work. Examination reveals a mass in the right abdomen, and CT scan demonstrates a mass in the right kidney. Which of the following is true about the presentation of renal cell carcinoma? A About 50% of patients will have hypercalcemia at presentation B Most patients will have erythrocytosis at presentation C Most patients will have hypertension at presentation D Most patients will be anemic at presentation E Only about 15% of patients will present with the classic triad of gross hematuria, flank pain, and a palpable mass

E. Only about 15% of patients will present with the classic triad of gross hematuria, flank pain, and a palpable mass

A 31-year-old woman presents with recurrent episodes of palpitations, tachycardia, dyspnea, and impending dread that began shortly after the birth of her first child. She was diagnosed with panic disorder after the first couple of episodes. Which is the best choice of drug for the sustained treatment of her condition? A. Lorazepam B. Hydroxyzine C. Buspirone D. Phenobarbitol E. Paroxetine

E. Paroxetine - Paroxetine is an SSRI which is 1st line tx for this

What is the earliest visible sign of an oral squamous cell carcinoma in situ? A raised nodular lesion with a pearly margin B. A submucosal mass C. Leukoplakia D. Lichen planus E. Persistent erythema

E. Persistent erythema

A 30-year-old man presents with severe indigestion of 1-month duration as his chief complaint. He has tried over-the-counter antacids for the last 2 weeks to help relieve the pain with only mild relief. He described the pain as a dull and aching, "hunger-like pain." He also states that a few hours after eating, the pain lessens and then will worsen a few hours later. His physical examination is unremarkable, except with epigastric pain on deep palpation of the abdomen. His stool guaiac test is negative. Which of the following therapeutic regimens would you prescribe? A H2-blocker therapy and bismuth therapy B Dietary modifications C Continue antacids and modify diet D Stress reduction, bismuth therapy, and dietary modifications E Proton pump inhibitor therapy, dietary modifications, and stress reduction

E. Proton pump inhibitor therapy, dietary modifications, and stress reduction

65 yo male pt is c/o chest pain upon exertion. which of the following is most useful and least invasive test for distinguishing reversible from non-reversible ischemia? A. holter monitoring B. EKG C. coronary arteriography D. MUGA scan E. thallium stress test

E. thallium stress test

which of the following findings can be visualized on stress echo and constitutes as evidence of myocardial ischemia? A. mitral regurg. B. akinesis of left ventricular wall C. ejection fraction of 65% D. coronary artery atherosclerotic plaque E. transient ventricular wall motion abnormalities

E. transient ventricular wall motion abnormalities

A 21-year-old college student presents with acute onset of right-sided chest pain and dyspnea for two hours. He has no past medical history. His vital signs on admission include a respiratory rate of 35/min, blood pressure of 80/50 mmHg, and pulse of 120/min. Chest x-ray is unremarkable. Chest CT demonstrates a saddle pulmonary embolism. He improves with thrombolytics. However, he has persistent shortness of breath for one year after the pulmonary embolism. A chest CT (shown below) demonstrates bilateral occluding centrally located pulmonary emboli. A pulmonary arteriogram defines these with near occlusion of the right main pulmonary artery and involvement of left upper and lower lobe arteries. Echocardiogram demonstrates elevated right ventricular pressure with preserved right and left ventricular function. The best treatment to improve survival in this disease entity is: A Oxygen B Inferior Vena Cava (IVC) Filter C Pneumonectomy D Lung Transplantation E Pulmonary Thromboendartectomy

E. Pulmonary Thromboendartectomy

A 40-year-old man with no significant past medical history presents to your office in January complaining of a four day history of fevers, chills, myalgias, headache, productive cough, and mild sinus congestion. His physical examination reveals a temperature of 102.7° F, pulse 96/min, respiratory rate 20/min, and blood pressure 128/80. There is mild maxillary sinus tenderness. His oral cavity and oropharynx are clear. His tympanic membranes are pearly gray with normal light reflex. His chest is clear to auscultation. Cardiac exam unremarkable. The best course of therapy for this patient is: A. Admit to the hospital for antibiotics and intravenous fluids B. Treat with oral trimethoprim/sulfamethoxazole for 7 days C. Prescribe amantadine D. Prescribe zanamivir E. Recommend bedrest, analgesics, and topical decongestants only

E. Recommend bedrest, analgesics, and topical decongestants only - This pt has influenza infection. No evidence of a severe bacterial infection so antibiotics would not be indicated. - It is important to note that the flu can cause sinus congestion as well as rhinorrhea. - Amantadine and zanamivir are helpful within 48 hours of the initial symptoms and can shorten sx by 1 day.

The child in the attached photograph has a rare facial cleft. You are the child's primary care physician. How will you manage this patient? A Advise the parents there are no reconstructive procedures available B Refer the family for psychiatric counseling C Order a CAT scan of the brain D Advise waiting until five years of age to undergo surgery E Refer the patient to a craniofacial team

E. Refer the patient to a craniofacial team

Which of the following is true regarding Rh status in pregnancy? A Rh negative blood is more common in African-Americans than in Caucasians. B Asians have the highest incidence of Rh isoimmunized pregnancies. C A primiparous patient with Rh negative blood should be counseled regarding the risk that her fetus will suffer from erythroblastosis fetalis. D A patient who has delivered a previous child with erythroblastosis fetalis should receive RhoGAM as soon as her next pregnancy is diagnosed. E RhoGAM should be prescribed to an Rh negative patient within 72 hours of delivery only if her baby is proven to be Rh positive.

E. RhoGAM should be prescribed to an Rh negative patient within 72 hours of delivery only if her baby is proven to be Rh positive.

A 29-year-old female presents 7 months postpartum to her primary care provider. She complains of 3 weeks of worsening pain with movement of the thumb. She states initially she noticed this pain only when picking up her infant, but that she has noticed it affecting more activities over time. On physical exam, there is a positive Finkelstein test. What other physical exam finding would support a diagnosis of DeQuervain's tenosynovitis? A Trigger digit B Limited extension of the wrist C Snuffbox tenderness D Thenar atrophy E Thickening over the radial styloid

E. Thickening over the radial styloid

32 yo former smoker presents with recent onset of intermittent chest pain which she describes as "knife-like" and occurs with deep breathing or coughing. She has no tenderness to palpation of her chest wall. Which of the following is the most likely dx? A. angina B. costochrondritis C. acute pancreatitis D. pleural effusion E. pleuritis

E. pleuritis

34 yo sexually active female presents to gyno for annual exam. Prior to exam, the PA notes her BP 180/125. fundoscopic, cardiac, and neurologic exam all normal. on auscultation of abdomen, a diastolic bruit is heard 1 cm superiolateral to the R of the umbilicus. What is the most likely secondary cause for her HTN? A. abdominal aortic aneurysm B. chronic pyelonephritis C. oral contraceptives D. polycystic renal disease E. renovascular disease

E. renovascular disease

64 yo male with a 10 year hx of HTN presents to your office complaining of dyspnea. You order pulmonary function tests, and he is found to have decreased TLC, decreased FVC, and increased FEV1/FVC. Which is the most likely dx? A. sarcoidosis B. small cell carcinoma of the lung C. cor pulmonale D. emphysema E. silicosis

E. silicosis

76 yo male presents with a productive cough. He reports exposure to a person with TB and your patient has a pmhx of a positive PPD. Which of the following is the best test for the definitive dx of TB in this patient? A. repeat PPD B. CXR C. blood culture D. direct florescent antibody test E. sputum culture

E. sputum culture

60 yo with squamous cell carcinoma of left lower lobe of lung. 2 cm lesion in diameter, centrally located, with no local extension. There is no lymphadenopathy or distant mets. Which is the best tx option? A. chemotherapy B no further tx C. both radiation and chemotherapy D. radiation therapy E. surgical resection

E. surgical resection

what is the preferred tx for a pituitary adenoma?

Transsphenoidal adenoma resection

what is the most important modifiable risk factor for avascular necrosis?

avoid alcohol

murphys sign is for appendix or gallbladder?

gallbladder

synovial fluid examination of gout would show what?

negatively birefringent, needle-shaped crystals under polarized light microscopy.

what types of patients is meningitis caused by Listeria monocytogenes seen in?

newborns, the elderly, the debilitated, diabetics, and alcoholics.

what medication can quickly lower highly elevated triglycerides

niacin (nicotinic acid)

first line tx for cushings disease

oral prednisone

what is the gold standard for dx a pulmonary embolism?

pulmonary angiography

is myxedema too much or too little thyroid hormone?

too little

what is the tx for lichen planus?

topical glucocorticoids for cutaneous lesions. Add triamcinolone for mucosal lesions

tx for scabies

topical permethrin

tx for Avascular necrosis

total hip replacement


Related study sets

Ch. 3 Assessing the Internal Environment of the Firm

View Set

Principals and Practice - Math and Chapter 9

View Set

Biology 101G Ch.1 The Process of Science

View Set

EOC Review Study Guide Questions, Semester 1 & 2

View Set

Praxis 5621 Principles of Learning and Teaching

View Set

Psychological Disorders 3360 - Final Fall 2021

View Set

NS3-M1U1C4 - U.S. Strategy & the Navy

View Set